SlideShare une entreprise Scribd logo
1  sur  39
Télécharger pour lire hors ligne
1.00       Introduction

A branch of mathematics where we count number of objects or number of ways of doing a particular job without
actually counting them is known as combinatorics and in this chapter we will with elementary combinatorics.
For example, if in a room there are five rows of chairs and each row contains, seven hours, then without counting
them we can say, total number of chairs is 35. We start this chapter principle of product or fundamental principle of
counting.FUNDAMENTAL PRINCIPLE OF COUNTING


 1.01       Fundamental Principal Of Counting

MULTIPLICATION PRINCIPLE OF COUNTING
If a job can be mm ways, and when it is done in any one of these ways another job be done in n, then both the jobs
together can be done in mn ways. The rule can be extended to move number of jobs.
Illustration 1.   A college offers 7 courses in the morning and 5 in the evening. Find the possible number of choices
                  with the student if he wants to study one course in the morning and one in the evening.

Solution:         The student has seven choices from the morning courses out of which he can select one course in 7
                  ways. For the evening course, he has 5 choices out of which he can select one in 5 ways. Hence the
                  total number of ways in which he can make the choice of one course in the morning and one in the
                  evening = 7 x 5 = 35.
Illustration 2.   A person wants to go from station A to station C via station B. There are three routes from A to B
                  and four routes from B to C. In how many ways can he travel from A to C?

Solution:         A         B in 3 ways
                  B    C in 4 ways
                  A     C in3x4 =12ways
Remark:           The rule of product is applicable only when the number of ways of doing each part is independent of
                  each other i.e. corresponding to any method of doing the first part, the other part can be done by any
                  method

Illustration 3.   How many (1) 5— digit (ii) 3 digit numbers can be formed by using 1, 2, 3, 4, 5 without repetition of
                  digits.

Solution:         (i) Making a 5-digit number is equivalent to filling 5 places.
                  Places:
                                                             1          2          3          4        5
                  Number of Choices:                         5          4          3          2   1
                  The first place can be filled in 5 ways using anyone of the given digits.
The second place can be filled in 4 ways using any of the remaining 4 digits. Similarly, we can fill
                  the 3rd, 4th and 5th place. No. of ways of filling all the five places = 5 x 4 x 3 x 2 x 1 = 120        120
                  5-digit numbers can be formed.
                  (ii) Making a 3-digit number is equivalent to filling 3 places.

                  Places: l                                        1          2            3


                  Number of Choices:                              5            4           3
                  Number of ways of filling all the three places = 5 x 4 x 3 = 60 Hence the total possible 3-digit
                  numbers = 60.
Illustration 4.   There are 10 steamers plying between Liverpool and Dublin; in how many ways can a man go from
                  Liverpool to Dublin and return by a different steamer?

Solution:         There are ten ways of making the first passage; and with each of these there is a choice of nine ways
                  of returning (since the man is not to come hack by the same steamer); hence the number of ways of
                  making the two journeys is 10 x 9, or 90. This principle may easily be extended to the case in which
                  there are more than two operations each of which can be performed in a given number of ways.

  Addition Principle :
If one experiment has n possible outcomes and another has m possible outcomes, then there are (m + n) possible
outcomes when exactly one of these experiments is performed. In other words, if a job can be done by n methods and
by using the first method, can be done in a1 ways or by second method in 2 ways and so on . . . by the nth method in a
ways, then the number of ways to get the job done is (a1       a .......... an ) .

Illustration 5.   A train is going from Cambridge to London stops at nine intermediate stations. Six persons enter the
                  train during the journey with six different tickets. How many different sets of tickets they have had?

Solution:         For S1 ,9 different tickets available, one for each of the remaining 9 stations; similarly at S 2 . 8

                  different tickets available; and so on.




                  Hence, it is clear, that total number of different tickets =9+8+7+6+5+4+3+2+1=45
                  Hence, the six different tickets must be any six of these 45; and there evidently as mar different sets
                  of 6 tickets as there are combinations of 45 things taken 6 at a time.
Illustration 6.   A college offers 7 courses in the morning and 5 in the evening. Find the number J of ways a student
                  can select exactly one course, either in the morning or in the evening.

Solution:         The student has seven choices from the morning courses out of which he can select one course in 7
                  ways. For the evening course, he has 5 choices out of which he can select one course in 5 ways.
Hence he has total number of 7 + 5 = 12 choices.
Illustration 7.   A person wants to leave station B. There are three routes from station B o A and J four routes from B
                  to C. In how many ways can he leave the station B.

Solution:         B           A in 3 ways
                  B        C in 4 ways
                          He can leave station B in 3 + 4 = 7 ways.


    1.02     Permutations (Arrangement Of Objects)

The number of permutations of n objects, taken r at a time, is the total number of arrangements of r objects, selected
from n objects where the order of the arrangement is important.
Without repetition:
(a) Arranging n objects, taken r at a time is equivalent to filling r places from n things.
r-places -
                              1             2         3          4                                    r


No. of choices-           n           n-1           n-2        n-3                           n-(r-1)

The number of ways of arranging = the number of ways of filling r places
=n (n—1) (n—2) (n—r+1)
    n(n 1)(n 2)...................(n r 1((n r )!)                  n!      n
=                                                                              Pr
                      (n r )!                                    (n r )!
                                                                                        n
(b) The number of arrangements of n different objects taken all at a time                   Pn   n!
With repetition: (a) The number of permutations (arrangements) of n different objects, taken r at a time, when each
object may occur once, twice, thrice .... up to r times in any arrangement
= The number of ways of filling r places where each place can be filled by any one of n objects.
r-places -
                              1             2         3          4                                    r


No. of choices-       n           n             n         n                                           n
                                                                                    r
The number of permutations = the number of ways of filling r places = ( n )
(b) The number of arrangements that can be formed using n objects out of which p are identical (and of one kind), q
                                                                                                             n!
are identical (and of another kind), r are identical (and of another kind) and the rest are distinct is
                                                                                                          p !q !r !
Illustration 8.   How many 7- letter words can be formed using the letters of the words?
                  (a)      BELFAST                    (b)     ALABAMA
Solution:         (a) BELFAST has all different letters.
                  Hence the number of words= 7 P7 = 7! = 5040

                  (b) ALABAMA has 4 A’s but the rest are all different. Hence the number of words
                  that can be formed is 7!/4!= 7 x 6 x 5 = 210.
Illustration 9.
                  a.   How many anagrams can be made by using the letters of the word HINDUSTAN.
                  b.   How many of these anagrams begin and end with a vowel.
                  c.   ln how many of these anagrams, all the vowels come together.
                  d. In how many of these anagrams, none of the vowels come together.
                  e.   In how many of these anagrams, do the vowels and the consonants occupy the same relative
                       positions as in HINDUSTAN.

Solution:         (a) The total number of anagrams
                  = Arrangements of nine letters taken all at a time = 9! /2! = 181440.
                  (b) We have 3 vowels and 6 consonants, in which 2 consonants are alike. The first place can be filled
                  in 3 ways and the last in 2 ways. The rest of the places
                  can be filled in 7! / 2! ways.
                  Hence the total number of anagrams 3 x 2 x 7! / 2! = 15120
                  (c) Assume the vowels (I, U, A) as a single letter. The letters (IUA) H, D, S, T, N,
                  N can be arranged in 7! / 2! ways. Also IUA can be arranged among themselves in 3! = 6 ways.
                  Hence the total number of anagrams = 7! / 2! x 6 = 15120
                  (d) Let us divide the task into two parts. In the first, we arrange the 6 consonants
                  as shown below in 6! / 2! ways.
                  x C x C x C x C x C x C x (C stands for consonants and x stands for blank spaces in between them)
                  Now 3 vowels can be placed in 7 places (in between the consonants) in 7 P3

                  7!
                  — =210 ways.
                  4!
                  Hence the total number of anagrams =6! / 2! x 210 = 75600.
                  (e) In this case, the vowels can be arranged among themselves in 3! = 6ways. Also, the consonants
                  can be arranged among themselves in 6! / 2! ways.
                  Hence the total number of anagrams 6! / 2! x 6= 2160.
Illustration 10. How many 3 digit numbers can be formed using the digits 0, 1, 2,3,4,5 so that (a) digits may not be
                  repeated (b) digits may be repeated.

Solution:         (a) Let the 3-digit number be XYZ
                  Position (X) can be filled by 1, 2,3,4,5 but not 0. So it can be filled in 5 ways.
Position (Y) can be filled in 5 ways again. (Since 0 can be placed in this position).
                 Position (Z) can be filled in 4 ways.
                 Hence by the fundamental principle of counting, total number of ways is
                 5 x 5 x 4 = l00ways.
                 (b) Let the 3 digit number be XYZ
                 Position (X) can be filled in 5 ways
                 Position (Y) can be filled in 6ways.
                 Position (Z) can be filled in 6 ways.
                 Hence by the fundamental principle of counting, total number of ways is
                 5 x 6 x 6 = 180




 1.03       Important Results

Number of permutations under certain conditions:
• Number of permutations of n different things, taken r at a time, when particular thing s to be always included in
   each arrangement is r n 1 Pr   1

• Number of permutations of n different things, taken r at a time, when a particular thin; is never taken in any
   arrangement is n 1Pr .

• Number of permutations of n different things, taken r at a time, when m particular things are never taken in any
                    n m
   arrangement is         Pr
   Number of permutations of n different things, taken all at a time, when m specific things always come together is
   (m!) (n — m + 1)!.
• Number of permutations of n different things, taken all at a time, when m specified things never come together is
   n! — m! (n — m + 1)!
Illustration 11. There are m men and n monkeys (n > m). if a man have any number of monkeys .in how many ways
                 may every monkeys have a master?

Solution:        The first monkey can select his master by m ways and after that the second monkey can select his
                 master again by m ways, so can the third. And so on,
                                                                                          n
                 hence all monkeys can select master = m x m x m…up to n times = m ways

Illustration 12. Find the number of ways in which we can arrange four letters of the word MATHEMAT.

Solution:        Letters of the word MATHEMATICS are (M, M), (A, A), (T, T), H, E, I, C and S, making eight
                 distinct letters. We can choose four out of them in 8C4    70 ways, and arrange each of these sets of
                 four in 4! = 24 ways, yielding (70) (24) = 1680 arrangements. Second, we can choose one pair from
among the three identical letter pairs, and two distinct letters out of the remaining seven in
                   3      7
                    C1     C2      3    7 6 /2        63 ways. The letters so obtained can be arranged in 4! /2! = 12
                 ways, so the number of arrangements in this case is (63) (12) = 756. Finally, we can choose two pairs
                 out of the three identical letter pairs. This can be done in 3C2 ways and the letters obtained can be

                 arranged in 4! /2! 2! = 6 ways, so that the number of arrangements in this last case is (3) (6) = 18.
                 Hence the total number of arrangements is 1680 + 756 + 18 = 2454.



 1.04       Cicular Permutations
In the event of the given n things arranged in a circular or even elliptical permutation — and in this case the first and
the last thing in the arrangement are indistinguishable — the number of :―permutations is (n—1)!.
For example is 20 persons are circularly arranged, the number of arrangements is 19!


       If positions on a circular arrangement are numbered, then can it be treated as linear arrangement?

however, in the case of circular permutations wherein clockwise and anticlockwise orders need not be differentiated
— as in the case of differently coloured beads or different flowers made in to a garland, the number of permutations is

   n 1!
        where the number of beads or flowers is taken as n. In details the concept is explained as follows:
    2

ARANGEMENTS AROUND A CIRCULAR TABLE

Consider five persons A, B, C, D, E be seated on the circumference of a circular table in Order which has no head
now, shifting A, B, C, D, E one position in anticlockwise direction we will get arrangements as shown in following
figure: We observe that arrangements in all figures are different. Thus, the number of circular permutations of n
different things taken all at a time is (n — 1)!, if clockwise and anticlockwise orders are taken as different.




Illustration 13. 20 persons were invited to a party. In how many ways can they and the host be seated at a circular
                 table? In how many of these ways will two particular persons be seated on either side of the host?

Solution:        1st part: Total persons on the circular table = 20 guest + 1                                            host
= 21
                 they can be seated in (21 — 1)! = 20! ways.
2nd part: After fixing the places of three persons
                 (1 host + 2 persons). Treating (1 host + 2 person) = 1 unit,
                 so we have now ((remaining 18 persons + 1 unit) = 19) and
                 the number of arrangement will be (19 — 1)! = 18! also these
                 two particular person can be seated on either side of the host
                 in 2! ways. Hence, the number of ways of seating 21 persons
                 on the circular table such that two particular persons be seated
                 on either side of the host= 18! x 2! =2x18!




 1.05       Arrangements Of Beads Or Flowers (All Different) Around A Circular Necklace Or Garland

Consider five beads A, B, C, D, E in a necklace or five flowers A, B, C, D, E in a garland etc. If the necklace or
garland on the left is turned over we obtain the arrangement on the right. i.e., anticlockwise and clockwise order of
arrangement is not different we will get arrangements as follows:
We see that arrangements in above figures are not different.
                                                                                        1
Then the number of circular permutations of n different things taken all at a time is     (n 1)! if clockwise and
                                                                                        2
anticlockwise orders are taken as not different.




Illustration 14. Consider 21 different pearls on a necklace. How many ways can the pearls be placed in on this
                 necklace such that 3 specific pearls always remain together?

Solution:        After fixing the places of three pearls. Treating 3 specific pearls = 1 units, so we have now 18 pearls
                 + 1 unit = 19 and the number of arrangement will be (19 — 1)! = 18! also, the number of ways of 3
                 pearls can be arranged between themselves is 3! 6. Since, there is no distinction between the
                                                                                                            1
                 clockwise and anticlockwise arrangements. So, the required number of arrangements =          18!. 6 = 3
                                                                                                            2
                 (18!).
NUMBER OF CIRCULAR PERMUTATIONS OF n DIFFERENT THINGS TAKEN r AT A TIME

CASE I -         If clockwise and anticlockwise orders are taken as different, then the required number of circular
                                      n
                                          Pr        n
                 permutations=               or         Cr         (r 1)!
                                          r

Illustration 15. In how many ways can 24 persons be seated round a table, if there are 13 seats?

Solution:        In case of circular table the clockwise and anticlockwise order are different, then the required number
                                                        24
                                                             P13       24!
                 of circular permutations =
                                                            13       13 11!
Case II -        If clockwise and anticlockwise orders are taken as not different, then the required number of circular
                                      n                 n
                                           Pr                Cr     (r 1)!
                 permutations =               or
                                          2r                       2



Illustration 16. How many necklace of 12 beads each can be made from 18 beads of various colours?

Solution:        In the case of necklace there is not distinction t between the clockwise and anticlockwise
                 arrangements, then the required number of circular permutations
                  18
                    P12        18!           18 17 16 15 14 13!                     119 13!
                  2 12        24 6!           6 5 4 3 2 1 24                           2
Illustration 17. In how many ways 10 boys and 5 girls can sit around a circular table so that no two girls sit together.

Solution:        10 boys can be seated in a circle in 9! ways. There are 10 spaces in between the boys, which can be
                                              10                                                         10
                 occupied by 5 girls in            P5 ways. Hence total numbers of ways are 9!                P5 .or 9! 10 C5 (5!) .

Illustration 18. If n distinct objects are arranged in a circle, show that the number of ways of selecting three of these

                                                                                       n(n 4)(n 5)
                 n things so that no two of them is next to each other is
                                                                                            6

Solution:        Let the n things be x1 , x2 ,..........................xn . The first choice may be one of these n things; and,

                 this is done in n C1 ways.

                 Suppose x1 is the one chosen; the next two may be chosen — excluding x1 and, the two next to x1 ,
                                                                              n 3                         n 3
                 namely x 2 , x n —from the remaining (n —3) in                     C 2 ways. Of these          C 2 there are (n — 4)
                 selections when the second two chosen are next to each other, like
                  x3 x4 .x4 x5 ,..........................xn 2 xn 1 .
                 the number of ways of selecting the second two after x1 is chosen, so that the two are not next to
                 each other is
n 3                          (n 3)(n 4)                         (n 4)(n 5)
                                  C2         (n 4)                  (n 4)
                                                            1.2                                 2
                            The two objects can be relatively interchanged in 2 ways. Further the order of the choice of the three
                            is not to be considered. Hence the number of ways of choice of the three is
                            n(n 4)(n 5) 2!                 n(n 4)(n 5)
                                 2      3!                      6

Illustration 19. 2n persons are to be seated n on each side of a long table. r(<n) particular persons desire to sit on one
                            side; and s(<n) other persons desire to sit on the other side. In how many ways can the persons be
                            seated?

Solution:                   For the side where r persons desire to sit, we need (n — r) more persons. This (n —r) may be chosen
                            from (2n — r — s) in          (2 n r s )
                                                                       Cn   r   ways. Automatically the remaining (n — s) person go to the other
                                                                                                             (2 n r s )
                            side where already there are s desirous of seating. Thus there are                            Cn   r   ways of distributing n

                            persons for each side providing for the restriction of on one side and s on the other side.
                            n persons on each side can be permuted in n seats in n! ways. The number of ways of seating the 2n
                            persons, n on each side, is therefore
                            (2 n r s )               2
                                         Cn   r   n! S


    1.06               Combinations

Each of different grouping or selections that can be made by some or all of a number of given things without
considering the order in which things are placed in each group, is called COMBINATIONS


                             Can combinations be considered for both non-identical and identical objects?


COUNTING FORMULAE FOR COMBINATIONS

The number of combinations (selections or groupings) that can be formed from n different objects taken r at a time is
denoted by n C r and its value is equal to
                                                                n
n                   n!                                              Pr !
    Cr                     0             r    n as n Cr
                  n r !r !                                          r!
as in a permutation the arrangement of r selected objects out of n, is done in r! ways and in combination arrangement
in a group is not considered.
In particular
        n
             C0    n
                       Cn     1 i.e. there is only one way to select none or to select all objects out of n distinct objects.
   n
        C1   n There are n ways to select one thing out of n distinct things.
                                           n   n
   n
        Cr   n
                 Cn    r    therefore Cx           Cy    x       y or x y           n
                                                             n
 If n is odd then the greatest value of n C r is Cn                  1   or n Cn   1
                                                                  2             2

                                                             n
 If n is even then the greatest value of n C r is Cn
                                                                  2

                      If the number of ways of selection of r objects out of 2n objects is maximum then,
                                                    What can be the possible value of r?


Illustration 20. Six X’s have to be placed in spaces on the adjoining Figure so that each row contains at least one X.
                       In how many different ways this can be done?




Solution:              If there be no restriction on the placing of the) (s, the number of ways is 8 C6   8
                                                                                                              C2   28 these
                       there are two ways in which the X; s can be placed; one, with the first row empty and the other with
                       the third row empty. These two cases only do not satisfy the condition.
                       So the number of ways = 28— 2 = 26.

IMPORTANT RESULTS OF COMBINATIONS (SELECTIONS)

 The number of ways in which r objects can be selected from n distinct objects if a particular object is always
                      n 1
    included is             Cr 1 .
 The number of ways in which r objects can be selected from n distinct objects if a particular object is always
                      n 1
    excluded is             Cr .
 The number of ways in which r objects can be selected from n distinct objects if m particular objects are always
    included is n n m Cr           m   .

 The number of ways in which r objects can be selected from n distinct objects if m particular objects are always
    excluded is n n m Cr .

Illustration 21. A lady desires to give a dinner party for 8 guests. In how many ways can the lady select guests for
                       the dinner from her 12 friends, if two of the guests will not attend the party together?

Solution:              The following three methods of approach are indicated.
                       (i) Number of ways of forming the party
12              10                    10
                 =        C8              C6 since              C6 is the number of ways of making up the party with both the specified
                 guests included.
                 =495—210=285
                                                                                               (OR)
                 (ii) Number of ways of forming the party
                 = Number of ways of forming without both of them
                 + Number of ways of forming with one of them and without the other
                     10
                 =        C8         2.10 C7              45 240                    285
                                                                                               (OR)
                 (iii) Split the number of ways of forming the party
                 = those with one of the two (say A) + those without A
                     10
                 =        C7          2.11 C8             120 165                    285

Note-         The number of ways in which r objects can be selected from n objects if m particular
                                                                     r                          r
                                                                          n m                        n m
                objects are identical is                                        Cr or                       Cr according as r                     m or r   m
                                                                    r 0                    r r m




Illustration 22. A bag contains 23 balls in which 7 are identical. Then find the number of ways of selecting 12 balls
                 from bag.

Solution:        Here n=23, p=7, r=12(r>p)
                                                                                                12
                                                                                                     16
                 Hence, required number of selections                                                     Cr
                                                                                               r 5


                     16              16             16              16          16         16              16         16
                          C5              C6              C7             C8          C9         C10             C11        C12
                          16          16                   16             16              16          16                16            16
                               C5              C6               C7             C8              C9          C10               C11           C12
                     17
                          C6         17
                                          C8        17
                                                          C10       17
                                                                          C12                                   n Cr        n
                                                                                                                                 Cr   1
                                                                                                                                           n 1
                                                                                                                                                 Cr
                     17
                          C11        17
                                           C9        17
                                                          C10        17
                                                                          C12                                  n Cr         n
                                                                                                                                 Cn   r
                          17              17                   17          17
                               C11             C12                  C9          C10
                     18              18              18              18
                          C12              C10             C6             C8




 1.07       Selection From Distinct / Identical Objects

SELECTION FROM DISTINCT OBJECTS
The number of ways (or combinations) of selection from n distinct objects, taken at least one of them is
n          n        n                                                     n
    C1         C2       C3   ..........................................       Cn   2n   1

Logically it can be explained in two ways, as one can be selected in n C1 ways, two in n C 2 ways and so on and by

addition principle of counting the total number of ways of doing either of the job is
n          n        n                     n
    C1         C2       C3   ..........       Cn

Also, for every object, there are two choices, either selection or non-selection. Hence total choices are 2 n . But this
also includes the case when none of them is selected. Therefore the number of selections, when at least one is selected

=        2n 1




                        It is allowed to select at most 5 things out of 11 different things. In how many ways
                                                       at least one of them can be selected?


Illustration 23. Given five different green dyes, four different blue dyes and three different red dyes, how many
                         combination of dyes can be chosen taking at least one green, one blue dye?

Solution:                Any one dye of a particular colour can be either chosen or not; and, thus there are 2 ways in
                         which each one may be dealt with.

                         Number of ways of selection so that at least one green dye is included                     25 1 31
                         (1 is subtracted to correspond to the case when none of the green dyes is chosen.)
                         A similar argument may be advanced in respect of other two colours also.

                         Number of combinations = 25                  1 24 1 23              31 15 8 3720


                                               SEL ECTION FROM IDENTICAL OBJECTS

1. The number of selections of r r                       n objects out of n identical objects is 1.
2. The number of ways of selections of at least one object out of n identical object is n.
3. The number of ways of selections of at least one out of a1 , a2 , a3 ...........an objects, where a1 are alike of one

         kind, a 2 are alike of second kind, and so on a n are alike of the nth kind, is
           a1 1 a2 1 ................... an 1                   1
4. The number of ways of selections of at least one out of a1                           a2   a3 ................... an   k objects, where a1 are
         alike of one kind a are alike of nth kind and k are distinct is

          a1 1 a2 1 ................... an 1 2k 1
Illustration 24. Find the number of combinations that can be formed with 5 oranges, 4 mangoes and 3 bananas when
                               it is essential to take
                               (i)        at least one fruit                          (ii) one fruit of each kind.

Solution:                      Here 5 oranges are alike of one kind, 4 mangoes are alike of second kind and 3 bananas are alike of
                               third kind
                               (i) The required number of combinations (when at least one fruit)
                               = (5 + 1) (4 + 1) (3 + 1)2°—i
                               = 120—1 =119
                               (ii) The required number of combinations (when one fruit of each kind)
                                  5             4            3
                                         C1         C1           C1     5 4 3               60

 1.08                   Divisors Of A Given Natural Number

Let n              N and n               P 1 .P2 2 .P3 3 .....................Pk k , where P , P2 , P3 ..............Pk are different prime numbers and
                                          1                                                 1


  1   ,    2   ,   3   ..............     k    are natural numbers then:

 the total number of divisors of N including 1 and n is

                   1      1,         2        2 ,        3       3 ..............       k     1
 the total number of divisors of n excluding 1 and n is

                   1      1,         2        2 ,        3       3 ..............       k     1     2
 the total number of divisors of n excluding exactly one out of 1 or n is

                   1      1,         2        2 ,        3       3 ..............       k     1     1
                                                                 1
 the sum of these divisors is                                         1   1,           2    1 ,..............      k   1
                                                                 2
          = P01 , P1 , P 2
                   1    1                       .......P 1
                                                        1             P02 , P21 , P22             .......P2   2
                                                                                                                  .......... P0 k , Pk1 , Pk 2 .......Pk   k




          (Use sum of G.P. in each bracket)
 the number of ways in which n can be resolved as a product of two factors is
          if n is not a perfect square
               1
                          1    1,          2        1 ,..............       k       1       1 if n is a perfect square
               2
 the number of ways in which is composite number n can be resolved into two factors which are relatively prime
                                                                                k 1
          (or co prime) to each other is equal to 2                                   where k is the number of different factors (or different primes) in n.
Illustration 25. If n = 10800then find the
                               (a)        total number of divisors of n
                               (b) the number of even divisors
(c)    the number of divisors of the form 4m + 2
                     (d) the number of divisors which are multiples of 15

                                     4
Solution:            n= 10800 = 2        33 52
                                                              a
                     Any divisor of n will be of the form 2       3b 5c where 0        a   4,0    b    3,0     c   2.
                     For any distinct choices of a, b and c, we get a divisor of n
                     (a) total number of divisors = (4 + 1) (3 + 1) (2 + 1) = 60
                     (b) for a divisor to be even, a should be at least one. So total number of even divisors
                           = 4(3 + 1) (2 + 1) = 48.
                     (c) 4m + 2 = 2(2m + 1). In any divisor of the form 4m + 2, a should be exactly 1. So number of
                           divisors of the form 4m + 2 = 1(3 + 1) (2 + 1) = 12.
                     (d) A divisor of n will be a multiple of 15 if b is at least one and c is at least one.
                           So number of such divisors = (4+1) x 3 x 2 = 30.

(8) Division into groups:

(i) The number of ways in which (m + n) different things can be divided into two groups which contain m and n
    things respectively is

     m n              (m n)!
           Cm nCn            ;m n
                       m!n !
Corollary : If m = n, then the groups are equal size. Division of these groups can be given by two types.
Type I : If order of group is not important:
The number of ways in which 2n different things can be divided equally into two groups is
 (2n)!
2!( n !) 2
Type II: If order of group is important:
The number of ways in which 2n different things can be divided equally into two distinct groups is
 (2n)!        2n !
        2
          2!
2!(n !)      ( n !) 2
(ii) The number of ways in which (m + n +p) different things can be divided into three groups which contain m , n
    and p things respectively is

     m n p                      (m n p )!
             Cm n p C n p C p                ;m       n   p
                                  m !n ! p !
Corollary : If m = n = p, then the groups are equal size. Division of these groups can he given by two types.
Type .I : If order of group is not important: The number of ways in which 3p different things can be divided equally
                         (3 p )!
into three groups is
                        3!( p !)3
Type II : If order of group is important : The number of ways in which 3p different things can be divided equally into
three distinct groups is
 (3 p)!        3 p!
         3
           3!
3!( p !)      ( p !)3
Note:
(1) If order of group is not important: The number of ways in which mn different things can be divided equally into m
                      mn !
    groups is
                   ( n !) m m !
(2) If order of group is important : The number of ways in which mn different things can be divided equally into m
                                         mn !       (mn)!
    distinct groups is                      m
                                                 m!
                                       (n !) m !    (n !) m
Illustration 26. In how many ways can a pack of 52 cards be equally among 4 players in order? . ‘

Solution.                  Here order ot group is important, then. the numbers of ways in which 52 different cards can be

                                                                 52!            (52)!
                           divided equally into 4 players is              4!
                                                               4!(13!)4        (13!) 4
Alternative method :

Each player will get 13 cards. Now first player can be given 13 cards out of 52 cards in 52C13 ways. Second player can
be given 13 cards out of remaining 39 cards (i.e. 52— 13 = 39) in 39C13 ways. Third player can be given 13 cards out
of remaining 26 cards (i.e., 39 — 13 = 26) in 26C13 ways and fourth player can be given 13 cards out of remaining 13
cards (i.e. 26— 13 = 13) in 13C13 ways.
Hence required number of ways
            52              39           26         13
                 C13             C13          C13        C13
              52!     39!     26!
                                    1
            13! 39! 13! 26! 13! 13!
             52!
                       4
             13!

Illustration 27. In how many ways can a pack of 52 cards be formed into 4 groups of 13 cards each ?

Solution.                  Here order of group is not important, then the number of ways in which 52 different cards can be
                           divided equally into 4 groups is
                             52!
                           4!(13!) 4

Alternative Method:
Each group will get 13 cards. Now first group can be given 13 cards out of 52 cards in C13 ways. Second group can be
given 13 cards out of remaining 39 cards (i.e. 52— 13 = 39) in 39C13 ways. Third group can be given 13 cards out of
remaining 26 cards (i.e., 39 — 13 = 26) in 26C13 ways and fourth group can be given 13 cards out of remaining 13
cards (i.e.,26 — 13 = 13) in 13C13 ways. But the all (four) groups can be interchanged in 4 ! ways. Hence the required
number of ways
            52              39        126          13
                 C13             C13        C13         C13
                                      4!
              52!     39!     26!       1
                                    1
            13! 39! 13! 26! 13! 13!    4!
              52!
                       4
                 13! 4!

Illustration28. In how many ways can a pack of 52 cards be divided in 4 sets, three of them having 17 cards each
                           and fourth just card?
                                                                                                                  (52)!
Solution.                  First we divide 52 cards into two sets which contains 1 and 51 cards respectively is
                                                                                                                  1!51!
                           Now 51 cards can be divided equally in three’ sets each contains 17 cards (Here order of sets is not
                                                   (51)!
                           important) in                   ways.
                                                  3!(17!)3
                           Hence the required number of ways
                                 (52)! (51)!
                                 1!51! 3!(17!)3
                                   (52)!                  (52)!
                                 1!3!(17!)3             (17!)3 3!
Alternative Method :
First set can be given 17 cards out of 52 cards in 52C17. Second set can be given 17 cards out of remaining 35 cards
(i.e. 52 — 17 = 35) in 35C17. Third set can be given 17 cards out of remaining 18 cards (i.e., 35 — 17 = 18) in 18C17
and fourth set can be given 1 card out of 1 card in 1C1. But the first three sets can be interchanged in 3 ! ways. Hence
the total number of ways for the required distribution
            52              35       1 18         1
                 C17             C17        C17    C1
                                     3!
               52     35!    18!       1
                                   1
            17! 35! 17! 18! 17! 1!     3!
              (52)!
            (17!)3 3!

Illustration29. In how many ways can 12 balls be divided between 2 boys, one receiving 5 and the other 7 balls?
                           Also in how many ways can these 12 balls be divided into groups of 5, 4 and 3 balls respectively ?

Solution.                  I Part: Here order is important, then the number of ways in which 12 different balls can be divided
                           between two boys which contains 5 and 7 balls respectively, is
                                 (12)!
                                       2! 1584
                                 5!7!
Alternative Method:
First boy can be given 5 balls out of 12 balls in 12C5. Second boy can be given 7 balls out of 7 balls (i.e. 12 — 5 = 7)
but there order is important (boys interchange by (2 types) then required no. of ways
                                12            7
                                     C5           C7   2!
                             12!
                                    1 2!
                            5! 7!
                            12! 2
                            5! 7!
                            12. 11. 10. 9. 8. 7! .2
                                5. 4. 3. 2. 1. 7!
                           1584
                       II part : Here order is not important then the number of ways in which 12 different balls can be
                       divided into three groups of 5, 4 and 3 balls respectively, is
                             (12)!
                                   27720
                            5!4!3!
Alternative Method:
First group can be given 5 balls out of 12 balls in C5 ways. Second group can be given 4 balls out of remaining 7 balls
(12 — 5 = 7) in 7C4 and 3 balls can be given out of remaining 3 balls in 3C3.
Hence the required number of ways (Here order of groups are not important)
             12        7             3
                  C5       C4            C3
               12!     7!
                          1
              5! 7! 4! 3!
                12!
              5! 4! 3!
(9) Arrangement in Groups:
(i) The number of ways in which n different things can be arranged into r different groups is
n r 1
        Pn or n !n 1 Cr     1

according as blank group are or are not admissible.

Illustration 30. In how many ways 5 different. balls can be arranged into 3 different boxes so that no box remains
                       empty?

Solution.              The required number of ways = 5!.5 1 C3    1   5!.4 C2    720

Alternative Method :

Each box must contain a least one ball since no box remains empty. Boxes can have balls in the following systems
3!
 All 5 balls can be arranged by 5 ! ways and boxes can be arranged in each system by
                                                                                                                 2!
                                                        3!    3!
 Hence required number of ways 5!                          5!    720
                                                        2!    2!
 (ii) The number of ways in which n different things can be distributed into r different groups is
 rn    r
           C1 (r 1) n         r
                                  C2 ( r    2) n .................... ( 1) r         1r
                                                                                          Cr   1


                                       x
 coefficient of xn in n !(e                 1) r
 Here blank groups are not allowed.

                                                   pr
 Note Coefficient of xr in e px
                                                   r!

Illustration 31. In how many ways 5 different balls can be distributed into 3 boxes so that no box remains empty?

 Solution.                The required number of ways
                                                             5                 5                    5
                                           35 – 3 C1 3 – 1        3
                                                                      C2 3 – 2 – 3 C3 3 – 3
                                           243 – 96 3 – 0
                                           150
                                                        OR
                          Coefficient of x in 5! (e x – 1) 3
                                                   5



                                           coeffi of x 5 in 5! e3 x – 3e 2 x       3e x – 1
                                                 35    25        1
                                           5!       –3       3
                                                 5!    5         5!
                                           35 – 3.25 3
                                           243 – 96 3
                                           150

 Alternative Method :
 Each box must contain at least one ball since no. box remains empty. Boxes can have balls in the following systems




 The number of ways to distribute the balls in I system = 5 C1                                 4
                                                                                                   C1   3
                                                                                                            C3
 The total number of ways to distribute 1, 1, 3 balls to the boxes

   5        4        3            3!
 = C1           C1       C3
                                  2!
   5 4 1 3 60
 and the number of ways to distribute the balls in II system
 = 5 C1    4
               C2    2
                         C2
The total number of ways to distribute 1, 2, 2 balls to the boxes

 5      4             2           3!
= C1        C2            C2
                                  2!
  5 6 1 3 90
The required number of ways = 60 + 90 = 150
(iii) The number of ways in which n identical things can be distributed into r different groups is
     n r 1                  n 1
             Cr   1   or         Cr    1

according as blank groups are or are not admissible.

Illustration 32. In how many ways 5 identical balls can be distributed into 3 different boxes so that no box remains
                           empty?

                                                               5 1
Solution.                  The required number of ways =             C3   1   = 4C2 = 6

Alternative Method:
Each box must contain at least one ball since no box remains empty. Boxes can have balls in the following systems




Here balls are identical but boxes are different the number of combinations will be I in each systems.
Required number of ways
       3!   3!
=1        1    6
       2!   2!
Illustration 33. Four boys picked up 30 mangoes. in how many ways can they divide them if all mangoes be
                           identical?

Solution.                  Clearly, 30 mangoes can be distributed among 4 boys such that each boy can receive any number of
                           mangoes.
                           Hence total number of ways
                           30 4 1
                                    C4     1   = 33C3 =5456

Illustration 34. Find the positive number of solutions of x + y + z + w = 20 under the following conditions:
                           (i)     zero values of x, y , z, w are included.                  (ii) zero values are excluded.
Solution.                  (i)     Since x + y + z + w = 20
                           Here x          0, y 0, z 0, w 0
                           The number of solutions of the given equation in this case is same as the number of ways of
                           distributing 20 things among 4 different groups.
                           Hence total number of solutions = 20       4 1
                                                                              C4   1
                                                                                       23
                                                                                            C3   1771
                           (ii) Since x+y+z+w = 20
Here                  x 1, y 1, z 1, w 1
                        or                   x –1 0, y –1 0, z –1 0, w –1 0
                        Let                   x1       x –1           x        x1 1
                                              y1       y –1           y        y1 1
                                              z1       z –1           z        z1 1
                                              w1       w –1            w        w1 1
                        Then from (1)
                                                           x1 1 y1 1 z1 1 w1 1 20
                                                                x1     y1       z1      w1      16
                         and                                x1         0, y1         0, z1       0, w1      0
                                                                                             16 4–1
                         Hence total number of solutions                                             C4–1
                                                           19    19. 18. 17
                                                                C3
                                                                  1. 2. 3
                                                          57. 17 969.
                         Alternative Method :
                         Part (ii ) :                 x        y z w                 20
                                                       x 1, y 1, z 1, w 1
                                                                                 20–1             19
                         Hence total no. of solutions                                   C4–1           C3
                                                                            969.

Illustration 35. How many integral solutions are there to x + y + z + t = 29, when x 1,y l, z 3andt 0?
Solution.       Since x + y + z + t = 29
                and x , y, z, t are integers
                                  x 1, y               2, z          3, t       0
                                 x – 1 0, y – 2                      0, z – 3 0, t               0
                 let             x1          x – 1, x2          y – 2, x3            z –3
                 or x    x1 1, y         x2        2, z         x3 3 andthen x1                  0, x2      0, x3   0, t   0
                 From (1),
                                      x1 1 x2               2 x3 3 t                       29
                                        x1        x2      x3 t            23
                                                                               23 4–1
                 Hence total number of solutions                                      C4–1
                                             26           26. 25. 24
                                                  C3                                 2600
                                                           1. 2. 3

Illustration 36. How many integral solutions are there to the system ‘of equations x1 +x2 +x3 +x4+x5 = 20 and x1 +x2
                = 15 when xk             0?
Solution.       We have x1 +x2 +x3 +x4+x5 = 20                                                                             ………………(i)
                and x1 + x2 = 15                                                                                               ………………(ii)
                then from (i) and (ii) we get two equation
x3     x4    x5       5
                 x1     x2    15

                 and given x1          0, x2       0, x3        0, x4         0 and x5        0
                                                                                             5 3–1
                 Then number of solutions of equation (iii)                                            C3–1
                                                           7              7. 6
                                                               C2                       21
                                                                          1. 2
                 and number of solutions of equation (iv) = 15 + 2— 1C2-1
                 = 16C1 = 16
                 Hence total number of solutions of the given system of equations = 21 x 16 =336.
Illustration 37. Find the number of non-negative integral solutions of 3x + y + z = 24.
Solution.        We have
                 let x = k
                 3x + y + z = 24 and given x                          0, y         0, z        0
                 y+z = 24—3k
                 Here                            24    24 – 3k            0
                 Hence 0           k       8
                 The total number of integral solutions of (i )is
                                                 24–3 k 2–1               25–3 k
                                                               C2–1                C1
                                                      25 – 3k
                 Hence the total number of solutions of the original equation
                                                       8                                8              8
                                                               25 – 3k          25          1– 3            k
                                                      k 0                            k 0           k 0


                               8.9
                 25. 9 – 3.                    225 – 108            117
                                2
                 (iv)        The number of ways in which n identical things can be distributed into r groups so that no
                             group contains less than l things and more than in things (l < m) is
                             ii. 1 1+1 1+2 in r
                                                                                                   l                                                  r
                             coefficient of xn in the expansion of x                                            xl   1
                                                                                                                           xl    2
                                                                                                                                       ...       xm

                                                                                                           lr            l 1 r               r
                             or coefficient of xn in the expansion of x                                         1- x m           1 x




Illustration 38. In how many ways can three persons, each throwing a single dia once, make a sum of 15?

Solution.        Numbers on the faces of the dia are 1, 2, 3, 4, 5, 6 (least number 1, greatest number 6)
3
                          Required number of ways                                      coefficient of x15 in x1                              x2   x3   x4   x5   x6
                                                                                                                                             3
                                           coeffi. of x15 in x 3 1 x x 2                                        x3        x4         x
                                                                                                                                         3
                                           coeffi. of x12 in 1 x x 2                                       x3        x4         x5
                                                                                               3              –3
                                           coeffi. of x12 in 1 – x 6                                1– x
                                           coeffi. of x12 in 1 – 3 x 6 3 x12 1 3 x 6 x 2 .... 28 x 6 ... 91x12 ...
                                           91 – 84 3                           94 – 84
                                           10

Illustration 39. In how many ways in which an examiner can assign 30 marks to 8 questions, giving not less than 2
                marks to any question.

Solution.       If examiner given marks any seven question 2 (each) marks then marks on remaining questions given
                by examiner = — 7 x 2+ 30 = 16
                If x are the marks assigned to ith question, then x1 + x2 + x3 + ... + x8 = 30 and 2                                                                      xi   l6fori =1,2,3
                8.
                Therefore the required number of ways
                                                    30               2             3                16 8
                     coefficient of                       in                             ...
                                          30               16                                  14 8
                     coeffi. of                    in           1              ....
                                          14                                           14 8
                     coeffi. of                    in 1                  ...
                                                                           8
                                          14              1–        15
                     coeffi. of                    in
                                                           1–
                                                                     –8
                     coeffi. of           14
                                                   in 1 –
                                          14                    8              9           2             21           14
                     coeffi. of                    in 1             C1             C2              ...        C13               ....
                      21                       21
                          C14                       C7

                     Note: Coefficient Of xr in the expansion of(l —x)-n is n + r - 1Cr

                (v) If a group has n things in which p are identical, then the number of ways of selecting r things
                           from a group is
                      r                        r
                           n– p                         n– p
                                  Cr or                        Cr according as r                         p or r            p.
                     r 0                  r r– p
Illustration 40. A bag has contains 23 balls in which 7 are identical. Then find the number of ways of selecting 12
                  balls from bag.

Solution.         Here n = 23,p=7,r=12 (r > p)
                                                                                             12
                                                                                                   16
                        Required number of selections                                                   Cr
                                                                                             r 5
                                          16            16              16           16                16        16                16             16
                                               C5             C6             C7              C8             C9        C10               C11            C12
                                            16               16                16             16                 16              16                   16         16
                                                   C5              C6               C7             C8                 C9              C10                  C11        C12
                                          17
                                               C6       17
                                                              C8        17
                                                                             C10        17
                                                                                             C12 n Cr                n
                                                                                                                              Cr –1     n 2
                                                                                                                                                 Cr
                                          17
                                               C11       17
                                                              C9        17
                                                                              C10       17
                                                                                             C12 n Cr                    n
                                                                                                                               Cn – r
                                            17                17                   17             17
                                                   C11             C12                  C9             C10
                                          18             18
                                               C12            C10
                                          18            18
                                               C6             C8

Deragements
If. n things are arranged in a row, the number of ways in which they can be deranged so that no one of them occupies
its original place is
        1 1 1 1                                    1
n! 1                ....................... ( 1) n
        1! 2! 3! 4!                                n!
No object goes to its scheduled place.
Remark: If r things goes to wrong place out of n things then (n — r) things goes to original place (Here r < n)
If Dn = No. of ways, if all n things goes to wrong place
and Dr = No. of ways, if r things goes to wrong place
If r goes to wrong places out of n, then (n — r) goes to correct places.
Then Dn = nCn - r Dr
                        1 1 1 1                                    1
Where Dr       r! 1                 ....................... ( 1) r
                        1! 2! 3! 4!                                r!

Illustration 41. A person writes letters to six friends and addresses the corresponding envelopes. in how many ways
                  can the letters he placed in the envelopes so that (i) at least two of them are in the wrong envelopes.
                  (ii) all the letters are in the wrong envelopes.
                                                                                                                                                                             6
                                                                                                                                                                                  n
Solution.          (i ) The number of ways in which at least two of them in the wrong envelopes                                                                                       Cn – r Dr
                                                                                                                                                                            r 2
                                               n                        n                     n                       n                      n
                                                   Cn – 2 D2                 Cn –3 D3              Cn – 4 D4              Cn –5 D5               Cn – 6 D6
                   Here n       6
                                                     1 1                                                1 1 1                                                1 1 1 1
                                  6
                                      C4 .2! 1 –                               6
                                                                                   C3 .3! 1 –                –                          6
                                                                                                                                            C2 .4! 1 –            –
                                                     1! 2!                                              1! 2! 3!                                             1! 2! 3! 4!
                                               1 1 1 1 1                                                                       1 1 1 1 1 1
                            6
                                C1 .5! 1 –          –     –                                             6
                                                                                                            C0 6! 1 –               –     –
                                               1! 2! 3! 4! 5!                                                                  1! 2! 3! 4! 5! 6!
15 40 135 264 265
                             719
                  (ii ) The number of ways is which all letters be placed in wrong envelopes
                                      1 1 1 1 1 1
                             6! 1 –        –     –
                                      1! 2! 3! 4! 5! 6!
                                   1 1    1        1
                             720    –        –120
                                   2 6    24      720
                             360 –120 30 – 6 1 265
Alternative Method:

(i) The number of all the possible ways of putting 6 letters into 6 envelopes is 6 !.
Number of ways to place all letters correctly into corresponding envelopes = 1
and Number of ways to place one letter is the wrong envelope and other 5 letters in the write envelope = 0
(: It is not possible that only one letter goes in the wrong envelop when if 5 letters goes in the right envelope, then
remaining one letter also goes in the write envelope)
Hence number of ways to place at least two letters goes in the wrong envelopes
=6!—0—l
=6! —1
= 720 —1 =719
(ii) The number of ways I letter in 1 address envelope so that one letter is in wrong envelope=0
(Because it is not possible that only one letter goes in the wrong envelope)

The number of ways to put 2 letters in 2 addressed envelopes so that all are in wrong envelopes
= The number of ways without restriction — The number of ways in which all are in correct envelopes — The number
of ways in which 1 letter is in the correct envelop
=2!—l—0
=2—1
=1 (2){from(1)}
The number of ways to put 3 letters in 3 addressed envelopes so that all are in wrong envelopes
= The number of ways without restriction — The number of ways in which all are in correct envelopes — The number
of ways in which 1 letter is in the correct envelope — The number of ways in which 2 letter are in correct envelope
3!—l—3C1 x l—0 (from(1),(2)} =2
(3C1 means that select one envelope to put the letter correctly)
The number of ways to put 4 letters in 4 addressed envelopes so that all are in wrong envelopes
The number of ways without restriction — The number of ways in which all are in correct envelopes — The number
of ways in which I letter is in the correct envelope — The number of ways in which 2 letter are in correct envelopes
— The number of ways in which 3 letters are in correct envelopes
4! –1 – 4 C1 2 – 4 C2 1 – 4 C3 0
  24 –1 – 8 – 6
  9
The number of ways to put 5 letters in 5 addressed envelopes so that all are in wrong envelopes
= The no. of ways without restriction — The no. of ways in which all are in correct envelopes — The no. of ways in
which 1 letter is in correct envelop — The no. of ways in which 2 letters are in correct envelopes — The-no, of ways
in which 3 letters are in correct envelopes — The no. of ways in which 4 letters are in correct envelopes
  5! –1 – 5C1 9 – 5C2 2 – 5C3 1 – 5C4 0
  120 –1 – 45 – 20 –10 – 0
  44
The number of ways-to put 6 letters in 6 addressed envelopes so that all are in wrong envelopes
= The no. of ways without restriction — The no. of ways in which all are in correct envelopes — The no. of ways in
which I letter is in the correct envelop — The no. of ways in which 2 letters are in correct envelopes — The no. of
ways in which 3 letters are in correct envelopes — The no. of ways in which 4 letters are in correct envelopes — The
no. of ways in which 5 letters are in correct envelopes.
  6! –1 – 6 C1 44 – 6 C2 9 – 6 C3 2 – 6 C4 1 – 6 C5 0
                                             {from (1), (2), (3), (4) & (5)}
  720 –1 – 264 –135 – 40 –15
  720 – 455    265.
Multinomial Theorem
(i) If there are 1 objects of one kind, m objects of second kind, n objects of third kind and so on; then the number of
ways of choosing r objects out of these objects (i.e.,l + m + n + ...) is the coefficient of xr in the expansion of
(l+x+x2+...+xl(l+x+x2+...+xm)( l+x+x2+...+xn)
Further if one object of each kind is to be included, there the number of ways of choosing r objects out of these objects
(i.e., 1 + m + n + ...) is the coefficient of x in the expansion of
(x + x2 ± x3 + ... x’) (x + + + ... X) (x + + x3 + ... + x)
(ii) If there are I objects of one kind, m object of second kind, n object of third kind and so on; then the number of
possible arrangements/permutations of r objects out of these objects (i.e. 1+ m + n + ...) is the coefficient of x in the
expansion of

        x x2               xl               x x2             xm   x x2              xn
r! 1          ............              1         ..........    1       ...........
        1! 2!              l!               1! 2!            m!   1! 2!             n!

Illustration 42. Find the number of Combinations and Permutations of 4 letters taken from the word
                  EXAMINATION.

Solution.         There are 11 letters A, A; I, I; N, N; E, X, M, T, O.
                                                                                      3         5
                  Then No. of combinations        coefficient of x 4 in 1 x x 2           1 x
                                                          2 A ' s, 2 I ' s, 2 N ' s,1E ,1X ,1M ,1Tand1O
                                                            3                  2                           5
                              coefficient of x 4 in   1 x       x6 3 1 x           x2 3 1 x x4 1 x
                                                            8            5                 7               6
                              coefficient of x 4 in   1 x       x6 1 x         3x 2 1 x         3x 4 1 x
                              8
                                  C4   0 3.7 C2   3
and No. of Permutations
                                                                            3              5
                                                4     x               x2          x
                             coefficient of x in 4! 1                           1
                                                      1!              2!          1!
                                                                           3
                                                                  x2                  5
                             coefficient of x 4 in 4! 1 x                       1 x
                                                                  2
                                                                  3        x6    3             2        3 4             5
                             coefficient of x 4 in 4!       1 x                    1 x             x2     x 1 x   1 x
                                                                           8     2                      4
                                                                  8        x6          5       3 2          7   3 4         6
                             coefficient of x 4 in 4!       1 x               1 x                x 1 x            x 1 x
                                                                           8                   2                4
                                  8            37       3
                             4!       C4   0     . C2
                                               2        4
                                  8.7.6.5      3 7.6    3
                             24                 .
                                  1.2.3.4      2 1.2    4
                             8. 7. 6. 5 6 3. 7. 6       6. 3      1680 756 18
Alternative Method :
There are 11 letters: A, A;I, I;N, N;E, X, M, T, O.
The following cases arise:
Case I : All letters different : The required number of choosing 4 different letters from 8 different (A, I, N, E, X, M, T,
0) types of the letters = 2454

8        8.7.5.6
    C4             70
         1.2.3.4
8
and No. of Permutations = 8P4 = 8.7.6.5 = 1680
Case II : Two alike of one type & Two alike of another type 2A’s, 2I’s, or 2I’s, 2N’s, or 2N’s, 2A’s.: This must be




= 18
Case III: Two alike and Two different: This must be 2A’s or 21’s or 2N’s and for each case 7 different letters.
= 756
From Case I,II and III,
The required No. of Combinations =70+3+63 and No. of Permutations = 1680 + 18 + 756 =2454.
How to find number of Solutions of the Equation
If the equation
             2    3     ... q         n                                                .....(1)
(i ) If zero included then no. of solutions of (1)
              coefficient of x n in 1 x x 2 ... 1 x 2                                                   x 4 ...

     1 x3    x 6 ... ... 1 x q            x 2 q ...
                                                            –1                     –1                      –1                      –1
              coefficient of x n in 1 – x                         1 – x2                    1 – x3              ... 1 – x q
(ii ) If zero exluded then the no. of solutions of (1)
             coefficient of x n in x x 2                         x 3 .... x 2                         x4        x 6 ... x 3                  x6       x 9 ...
                                                                                                                                    .... x q           x2q       ...
                                                                                  –1                  2 –1                 3 –1                     q –1
             coefficient of x n in x1               2 3 ... q
                                                                   1– x                 1– x                    1– x              ... 1 – x
                                      q q 1
                                 n–                                     –1                       –1                   –1                       –1
             coefficient of x           2
                                                    in 1 – x                  1 – x2                   1 – x3              ... 1 – x q

Illustration 43. Find the number of non negative integral solutions of x1 x2                                                                                     x3         4 x4    20 .

Solution.          Number of non negative integral solutions of the given equation
                                                                                                 –1                –1                –1                    –1
                                coefficient of x 20 in 1 – x                                           1– x                1– x              1 – x4
                                                                                                 –3                   –1
                                coefficient of x 20 in 1 – x 1 – x4
                                coefficient of x in (1 C1 x 4 C2 x 2
                                                 20       3                                                                 5
                                                                                                                                 C3 x 3        6
                                                                                                                                                   C4 x 4 ...          10
                                                                                                                                                                            C8 x8    ...   14
                                                                                                                                                                                                C12 x12 ..
                                           18
                                 ....           C16 x16 ...                  22
                                                                                  C20 x 20 ...)
                                                                                                                                                                1 x4          x8    x12     x16     x 20 ...
                                      6              10            14                  18              22
                                1         C4              C8            C12                 C16              C20
                                      6              10            14              18                 22
                                1         C2              C2            C2                  C2             C2
                                           6. 5                10. 9                    14. 13                   18. 17                   22. 21
                                1
                                           1. 2                1. 2                      1. 2                     1. 2                     1. 2
                                1 15 45 91 153 231
                                536
Illustration 44. Find the number of positive unequal integral solution of the equation x + y + z + w = 20;

Solution.          We have
                   x   y z w                20
                   assume x           y         z      w, Here x, y, z , w 1
                   now let x          x1 , y – x             x2 , z – y                 x3 and w – z                        x4
                         x      x1 , y          x1        x2 , z        x1         x2            x3 , w          x1        x2       x3         x4
                   from (1), 4 x1 3 x2                     2 x3         x4         20, then x1 , x2 , x3 , x4                            1
                       4 x1 3 x2               2 x3        x4          20                                                                                  ......(2)
                        Number of positive integral solutions of (2)
                                                                                              –1                   –1                     –1               –1
                         coefficient of x 20–10 in 1 – x 4                                            1 – x3               1 – x2              1– x
x6    x8     x10     x9   x4   x6   x8   x10   x7   x9    x10     x8    x10 )(1 x x 2
                                            x3   x4   x5   x6   x7    x8   x9    x10 )
                             1 1 1 1 1 1 1 1 1 1 1 1 1 1 1 1 1 1 1 1 1 1 1
                             23
Number of Rectangles and Squares
                                                                                n                                                n
(i)   Number of rectangles of any size in a square of n x n is                        r 3 and number of squares of any size is         r2
                                                                                r 1                                              r 1


                                                                                              np
(ii) In a rectangle of n x p (n <p) number of rectangles of any size is                          (n 1)( p 1) and number of squares of any
                                                                                               4
                n
      size is         (n 1 r )( p 1 r )
                r 1


Illustration 45. Find the number of rectangles excluding squares from a rectangle of size 9 × 6.

Solution.             Here n =6 and p =9.
                      No. of rectangles excluding square
                                            6
                          6.9
                              (6 1)(9 1) – (7 – r )(10 – r )
                           4               r 1
                                     6
                            945 –         (70 –17r r 2 )
                                    r 1

                            945 –154        791.
Exponent of Prime p inn!
Since exponent of prime p in n is denoted by Ep (n !). where p is a prime number and n is a natural number. Then the

                                                                                           n          n
last integer amongst 1, 2, 3, ...,(n — 1), n which is divisible by p is                      p, where   denotes the greatest integer.
                                                                                           p          p
 [ x]   x
                     E p (n !)    E p (1.2.3.....(n –1).n)

                                                            n
                     E p p.2 p.3 p.....(n –1) p.              p
                                                            p
                     because the remaining natural numbers from 1 to n are not divisible by p.
                         n                         n
                                 E p 1.2.3....
                         p                         p
                                                                            n
                      Now the last integer amongs 1, 2,3....,                 which is divisible by p is
                                                                            p
                        n. p         n         n                             n
                                                        E p p.2 p.3 p....       p
                         p           p2        p                             p2
                                                                                          n
                      because the remaining natural numbers from 1 to                       are not divisible by p.
                                                                                          p
                         n        n                          n
                                           E p 1.2.3....
                         p        p2                         p2
                 similarly, we get

                                 n        n            n            n
                  E p (n !)                                  ....
                                 p        p2           p3           ps
                 Where s is the largest natural number such that p s                  n     ps   1



Illustration 46. Find the exponent of 3 in 100!.

Solution:        In terms of prime factors 100 ! can be written as 2a. 3b 5c. 7d…………………….
                                               100           100         100          100
                 Now, E3 (100!)
                                                3             32          33           34
                 =33 + 11+3+1=48
                 Hence, the exponent of 3 in 100 ! is 48 ( 100 ! = 2a. 348 .5c .7d )
                                     33        33            33      33          33
                 E2 (33!)
                                     2         22            23      24          25
                 = 16+8+4+2+1 = 31
                 Hence, the exponent of 2 in 33 ! in 31. Now 33 ! is divisible by 231 which is also divisible by 219
                 largest value of n is 31.

Illustration 47. Find the number of zeros at the end of 100!?

Solution:        In terms of prime factors 100 ! can be written as 2a. 3b .5c. 7d
                                               100           100         100          100            100   100
                 Now, E2 (100!)
                                                2             22          23           24             25    26
50 25 12 6 3 1
                         97
                                               100   100
                         and E5 (100!)
                                                5     52
                         20 4
                         24
                         100! 297.3b.524.7 d ...
                         273.3b.(2 5) 24 .7 d ...
                         273.3b.(10) 24 .7 d ...
                      Hence number of zeros at the end of 100! is 24.
                                            OR
                      Exponent of 10 in 100! min (97, 24)        24.

Important Results to Remember
(i)     n straight lines are drawn in the plane such that no two lines are parallel and no three lines are concurrent.
        Then the number of parts into which these lines divide the

        plane is equal to 1+            n

Illustration 48. Six straight lines are drawn in the plane such that no two lines are parallel and no three lines are
                  concurrent. Then find the number of parts into which these lines divide the plane.

Solution.         No. of parts of the plane = 1 +          6

                         6.7
                  =1+        = 22
                         1.2
(ii)    The sum of the digits in the unit place of all numbers formed with the help of a1 ,a2………… an taken all at a
        time is
        = (n — 1)! (a1 ,a2………… an) (repetition of digits not allowed)

Illustration 49. Find the sum of the digits in the unit place of all numbers formed with the help of 3, 4, 5, 6 taken all
                  at a time.

Solution.         Sum of the digits in the unit place
                  = (4—1)!(3+4+5+6)
                  = 6.18 = 108.
(iii)   The sum of all digit numbers that can be formed using the digits a1 ,a2………… an (repetition of digits not
        allowed) is

                                                     (10n 1)
            (n 1)!(a1 a2 ............... an )
                                                        9
Illustration 50: Find the sum of all five digit numbers that can he formed using the digits 1, 2,3, 4 and 5 (repetition of
                  digits not allowed).

                                                               (105 1)
                                                                  9
Solution.                   Required sum= (5—1)! (1 +2+3+4+5)
                            = 24.15. 11111
                            = 3999960.

Alternative Method:

Since one of the numbers formed with the 5 digits a , b , c , d, e is
104a+ 103b + 102c+ l0d+e;
Hence 104a will occur altogether in 4 ! Ways similarly each of 104b, 104c, 104d, 104e will occur in 4 ! Ways. Hence, if
all the numbers formed with the digits be written one below the other, thus
104.a 103.b 102.c 10.d e
104.b 103.c 102.d 10.e a
104.c 103.d 102.e 10.a b
104.d 103.e 102.a 10.b c
104.e 103.a 102.b 10.c d
Hence the required sum
       4!     (a b c d e) (104 103 102 10 1)
       4!     (1 2 3 4 5) (11111)
       3999960.

(iv)          If there are n rows, I row has                1   squares. II row has           2   . squares, III row has           3   squares, and so on. If we

              placed          Xs in the squares such that each row contain at least one X. The number of ways = coefficient of

                  x in
   1
       C1 x        1
                       C2 x 2 ............   1
                                                 C 1x   1          2
                                                                       C1 x      2
                                                                                     C2 x 2 ............       2
                                                                                                                   C 2x     1
                                                                                                                                ( 3 C1 x       3
                                                                                                                                                   C2 x 2 .....
.......       3
                  C 3x 3)

Illustration 51. Six X’s have to be placed in the squares of the figure below, such that each row contains at least one
                            X. In how many different ways can this be done?




Solution:                   The required no. of ways
                            = coefficient of x6 in ( 2 C1 x            2
                                                                           C2 x 2 )( 4 C1 x   4
                                                                                                  C2 x 2   4
                                                                                                               C3 x 3   4
                                                                                                                            C4 x 4 )( 2 C1 x       2
                                                                                                                                                       C2 x 2 )
                            = coefficient of x3 in (2 + x)2 (4 + 6x + 4x2 + x3)
= coefficient of x3 in (4 + 4x + x2) (4 + 6x + 4x2 + x3)
                       = 4+16+6 = 26


Alternative Method
In the given figure there are 8 squares and we hay to place 6X ‘s this can be done in




                                  X                 X


                X                 X                 X                      X




   8        8          8.7
i.e. C6         C2            28 ways
                       1.2
But these include the possibility that either headed row or lowest row may not have any X. These two possibilities are
to be excluded. Required number of ways = 28 — 2 = 26




                                                 Solved Examples


Example 1: A letter lock consists of three rings each marked with 10 different letters. In how many ways is it
                     possible to make an unsuccessful attempt to open the block?

Solution:            Two rings may have same letter at a time but same ring cannot have two letters at time, therefore, we
                     must proceed ring wise. Each of the three rings can have any one of the 10 different letters in 10 ways.
                     Therefore Total number of attempts = 10 × 10 × 10 = 1000.
                     But out of these 1000 attempts only one attempt is successful.
                     Therefore Required number of unsuccessful attempts
                      = 1000 - 1 = 999.
Example 2: Find the total number of signals that can be made by five flags of different colour when any number of
                 them may be used in any signal.

Solution:
                 Case I :    When only one flag is used.
                             No. of signals made = 5P1 = 5.
                 Case II : When only two flag is used.
                             Number of signals made = 5P2 = 5.4 = 20.
                 Case III : When only three flags are used.
                              Number of signals is made = 5P3 = 5.4.3 = 60.
                 Case IV : When only four flags are used.
                              Number of signals made = 5P4 = 5.4.3.2 = 120.
                 Case V :     When five flags are used.
                              Number of signals made = 5P5 = 5! = 120.
                              Hence, required number = 5 + 20 + 60 + 120 + 120 = 325.
 Example 3: Prove that if each of the m points in one straight line be joined to each of the n points on the other
                 straight line, the excluding the points on the given two lines. Number of points of intersection of these
                 lines         is 1/4 mn (m-1(n-1).

Solution:        To get one point of intersection we need two points on the first line and two points on the second line.
                 These can be selected out of n-points in nC2 ways and for m points in mC2 ways.
                                                 2n 1            2n 1                      2n 1        2n 1
                 Therefore Required number = Now      C0                C2 n   1       1          C1          C2 n etc.........

                        m m 1   n n 1
                   =          .
                          2!      2!
                  = 1/4 m n (m - 1)(n - 1)
Example 4:       There are ten points in a plane. Of these ten points four points are in a straight line and with the
                 exception of these four points, no other three points are in the same straight line. Find
                 (i)     the number of straight lines formed.
                 (ii)    the number of triangles formed.
                 (iii) the number of quadrilaterals formed by joining these ten points.
Solution:
(i)   For straight line, we need 2 points
No. of point selected out           No. of points selected out           No. of straight line
of 4 collinear points               of remaining 6 points                formed
                                                                                   4
             0                                   2                                 C0 × 6C2 = 15
                                                                                   4
             1                                   1                                 C1 × 6C1 = 24
Permutation & combination
Permutation & combination
Permutation & combination
Permutation & combination
Permutation & combination
Permutation & combination

Contenu connexe

Tendances

G6 m3-c-lesson 15-t
G6 m3-c-lesson 15-tG6 m3-c-lesson 15-t
G6 m3-c-lesson 15-tmlabuski
 
G6 m3-c-lesson 14-t
G6 m3-c-lesson 14-tG6 m3-c-lesson 14-t
G6 m3-c-lesson 14-tmlabuski
 
Memetic Firefly algorithm for combinatorial optimization
Memetic Firefly algorithm for combinatorial optimizationMemetic Firefly algorithm for combinatorial optimization
Memetic Firefly algorithm for combinatorial optimizationXin-She Yang
 
Lesson 1 3 exponents
Lesson 1 3 exponentsLesson 1 3 exponents
Lesson 1 3 exponentsmlabuski
 
G6 m3-c-lesson 18-t
G6 m3-c-lesson 18-tG6 m3-c-lesson 18-t
G6 m3-c-lesson 18-tmlabuski
 
Linear algebra in the dirac notation
Linear algebra in the dirac notationLinear algebra in the dirac notation
Linear algebra in the dirac notationLucasOliveiraLima
 
Mathematics In Plain Sight
Mathematics In Plain SightMathematics In Plain Sight
Mathematics In Plain SightDavid Krueger
 

Tendances (8)

Tot d sokal
Tot d sokalTot d sokal
Tot d sokal
 
G6 m3-c-lesson 15-t
G6 m3-c-lesson 15-tG6 m3-c-lesson 15-t
G6 m3-c-lesson 15-t
 
G6 m3-c-lesson 14-t
G6 m3-c-lesson 14-tG6 m3-c-lesson 14-t
G6 m3-c-lesson 14-t
 
Memetic Firefly algorithm for combinatorial optimization
Memetic Firefly algorithm for combinatorial optimizationMemetic Firefly algorithm for combinatorial optimization
Memetic Firefly algorithm for combinatorial optimization
 
Lesson 1 3 exponents
Lesson 1 3 exponentsLesson 1 3 exponents
Lesson 1 3 exponents
 
G6 m3-c-lesson 18-t
G6 m3-c-lesson 18-tG6 m3-c-lesson 18-t
G6 m3-c-lesson 18-t
 
Linear algebra in the dirac notation
Linear algebra in the dirac notationLinear algebra in the dirac notation
Linear algebra in the dirac notation
 
Mathematics In Plain Sight
Mathematics In Plain SightMathematics In Plain Sight
Mathematics In Plain Sight
 

Similaire à Permutation & combination

Permutation and combinations
Permutation and combinationsPermutation and combinations
Permutation and combinationsRushabh Vora
 
Permutation Combination
Permutation Combination Permutation Combination
Permutation Combination RizwanManzoor15
 
Permutations and-combinations-maths
Permutations and-combinations-mathsPermutations and-combinations-maths
Permutations and-combinations-mathsMurugan Iron
 
Permutation & Combination
Permutation & CombinationPermutation & Combination
Permutation & CombinationPuru Agrawal
 
Permutation and combination
Permutation and combinationPermutation and combination
Permutation and combinationmanishkanwar
 
Lecture Week 17 which hleps in study for logic and
Lecture Week 17 which hleps in study for logic andLecture Week 17 which hleps in study for logic and
Lecture Week 17 which hleps in study for logic andmanishhmishra001
 
Combinations and permutations
Combinations and permutationsCombinations and permutations
Combinations and permutationsindu psthakur
 
Permutation combination
Permutation combinationPermutation combination
Permutation combinationlovemucheca
 
counting principle.ppt
counting principle.pptcounting principle.ppt
counting principle.pptRizaCatli2
 
Mathematical Statistics Homework Help
Mathematical Statistics Homework HelpMathematical Statistics Homework Help
Mathematical Statistics Homework HelpExcel Homework Help
 
Permutation in Discrete Mathematics- Piyush Bujade
Permutation in Discrete Mathematics- Piyush BujadePermutation in Discrete Mathematics- Piyush Bujade
Permutation in Discrete Mathematics- Piyush BujadetheLegendPiyush
 
A Solution Manual For A First Course In Probability
A Solution Manual For  A First Course In ProbabilityA Solution Manual For  A First Course In Probability
A Solution Manual For A First Course In ProbabilityMary Calkins
 

Similaire à Permutation & combination (20)

Permutation and combinations
Permutation and combinationsPermutation and combinations
Permutation and combinations
 
Permutation Combination
Permutation Combination Permutation Combination
Permutation Combination
 
Permutations and-combinations-maths
Permutations and-combinations-mathsPermutations and-combinations-maths
Permutations and-combinations-maths
 
PermutATIONS
PermutATIONSPermutATIONS
PermutATIONS
 
Permutation and combination
Permutation and combinationPermutation and combination
Permutation and combination
 
Permutation & Combination
Permutation & CombinationPermutation & Combination
Permutation & Combination
 
Permutation and combination
Permutation and combinationPermutation and combination
Permutation and combination
 
Lecture Week 17 which hleps in study for logic and
Lecture Week 17 which hleps in study for logic andLecture Week 17 which hleps in study for logic and
Lecture Week 17 which hleps in study for logic and
 
Bba ii-u1-p&c
Bba ii-u1-p&cBba ii-u1-p&c
Bba ii-u1-p&c
 
Combinations and permutations
Combinations and permutationsCombinations and permutations
Combinations and permutations
 
Permutation combination
Permutation combinationPermutation combination
Permutation combination
 
counting principle.ppt
counting principle.pptcounting principle.ppt
counting principle.ppt
 
Mathematical Statistics Homework Help
Mathematical Statistics Homework HelpMathematical Statistics Homework Help
Mathematical Statistics Homework Help
 
Permutation in Discrete Mathematics- Piyush Bujade
Permutation in Discrete Mathematics- Piyush BujadePermutation in Discrete Mathematics- Piyush Bujade
Permutation in Discrete Mathematics- Piyush Bujade
 
Mathematical Statistics Homework Help
Mathematical Statistics Homework HelpMathematical Statistics Homework Help
Mathematical Statistics Homework Help
 
Em08 ect
Em08 ectEm08 ect
Em08 ect
 
Permutations and combinations
Permutations and combinationsPermutations and combinations
Permutations and combinations
 
Permutations and combinations
Permutations and combinationsPermutations and combinations
Permutations and combinations
 
Permutations and combinations
Permutations and combinationsPermutations and combinations
Permutations and combinations
 
A Solution Manual For A First Course In Probability
A Solution Manual For  A First Course In ProbabilityA Solution Manual For  A First Course In Probability
A Solution Manual For A First Course In Probability
 

Plus de APEX INSTITUTE

IIT- JEE Main 2016 Paper solution
IIT- JEE Main 2016 Paper solutionIIT- JEE Main 2016 Paper solution
IIT- JEE Main 2016 Paper solutionAPEX INSTITUTE
 
IIT - JEE Main 2016 Sample Paper -5
IIT - JEE Main 2016 Sample Paper -5IIT - JEE Main 2016 Sample Paper -5
IIT - JEE Main 2016 Sample Paper -5APEX INSTITUTE
 
IIT - JEE Main 2016 Sample Paper 3
IIT - JEE Main 2016 Sample Paper 3IIT - JEE Main 2016 Sample Paper 3
IIT - JEE Main 2016 Sample Paper 3APEX INSTITUTE
 
IIT - JEE Main 2016 Sample Paper -4
IIT - JEE Main 2016 Sample Paper -4IIT - JEE Main 2016 Sample Paper -4
IIT - JEE Main 2016 Sample Paper -4APEX INSTITUTE
 
IIT- JEE Main 2016 Sample Paper-2
IIT- JEE Main 2016 Sample Paper-2IIT- JEE Main 2016 Sample Paper-2
IIT- JEE Main 2016 Sample Paper-2APEX INSTITUTE
 
IIT- JEE Main 2016 Sample Paper-1
IIT- JEE Main 2016 Sample Paper-1IIT- JEE Main 2016 Sample Paper-1
IIT- JEE Main 2016 Sample Paper-1APEX INSTITUTE
 
Crash-Course for AIPMT & Other Medical Exams 2016(Essentials heart)
Crash-Course for AIPMT & Other Medical Exams 2016(Essentials heart)Crash-Course for AIPMT & Other Medical Exams 2016(Essentials heart)
Crash-Course for AIPMT & Other Medical Exams 2016(Essentials heart)APEX INSTITUTE
 
Crash-Course for AIPMT & Other Medical Exams 2016Target pmt (2)
Crash-Course for AIPMT & Other Medical Exams 2016Target pmt (2)Crash-Course for AIPMT & Other Medical Exams 2016Target pmt (2)
Crash-Course for AIPMT & Other Medical Exams 2016Target pmt (2)APEX INSTITUTE
 
Crash-Course for AIPMT & Other Medical Exams 2016 (Essentials cockroach)
Crash-Course for AIPMT & Other Medical Exams 2016 (Essentials cockroach)Crash-Course for AIPMT & Other Medical Exams 2016 (Essentials cockroach)
Crash-Course for AIPMT & Other Medical Exams 2016 (Essentials cockroach)APEX INSTITUTE
 
Class X SA-II MATHEMATICS SAMPLE PAPER 2016
Class X SA-II MATHEMATICS SAMPLE PAPER 2016Class X SA-II MATHEMATICS SAMPLE PAPER 2016
Class X SA-II MATHEMATICS SAMPLE PAPER 2016APEX INSTITUTE
 
Class X SA-II SCIENCE SAMPLE PAPER 2016
Class X SA-II SCIENCE SAMPLE PAPER 2016Class X SA-II SCIENCE SAMPLE PAPER 2016
Class X SA-II SCIENCE SAMPLE PAPER 2016APEX INSTITUTE
 
Class X SA-II SCIENCE SAMPLE PAPER 2016
Class X SA-II SCIENCE SAMPLE PAPER 2016Class X SA-II SCIENCE SAMPLE PAPER 2016
Class X SA-II SCIENCE SAMPLE PAPER 2016APEX INSTITUTE
 
I.S.C. Class XII MATHEMATICS Sample Papers 2016
I.S.C. Class XII MATHEMATICS Sample Papers 2016I.S.C. Class XII MATHEMATICS Sample Papers 2016
I.S.C. Class XII MATHEMATICS Sample Papers 2016APEX INSTITUTE
 
I.S.C. Class XII Sample Papers 2016
I.S.C. Class XII Sample Papers 2016I.S.C. Class XII Sample Papers 2016
I.S.C. Class XII Sample Papers 2016APEX INSTITUTE
 
I.S.C. Class XII Sample Papers 2016
I.S.C. Class XII Sample Papers 2016I.S.C. Class XII Sample Papers 2016
I.S.C. Class XII Sample Papers 2016APEX INSTITUTE
 
Crash Course For IIT-Main sample paper 2016
Crash Course For IIT-Main sample paper 2016Crash Course For IIT-Main sample paper 2016
Crash Course For IIT-Main sample paper 2016APEX INSTITUTE
 
SUMMATIVE ASSESSMENT-II MATHS SAMPLE TEST
SUMMATIVE ASSESSMENT-II MATHS SAMPLE TEST SUMMATIVE ASSESSMENT-II MATHS SAMPLE TEST
SUMMATIVE ASSESSMENT-II MATHS SAMPLE TEST APEX INSTITUTE
 
Prospectus FOR IIT-JEE, AIPMT, NTSE, OLYMPIAD
Prospectus FOR IIT-JEE, AIPMT, NTSE, OLYMPIADProspectus FOR IIT-JEE, AIPMT, NTSE, OLYMPIAD
Prospectus FOR IIT-JEE, AIPMT, NTSE, OLYMPIADAPEX INSTITUTE
 
SUMMATIVE ASSESSMENT-II MATHS SAMPLE TEST
SUMMATIVE ASSESSMENT-II MATHS SAMPLE TEST SUMMATIVE ASSESSMENT-II MATHS SAMPLE TEST
SUMMATIVE ASSESSMENT-II MATHS SAMPLE TEST APEX INSTITUTE
 
AIPMT-2016 SAMPLE TEST PAPER-14
AIPMT-2016 SAMPLE TEST PAPER-14AIPMT-2016 SAMPLE TEST PAPER-14
AIPMT-2016 SAMPLE TEST PAPER-14APEX INSTITUTE
 

Plus de APEX INSTITUTE (20)

IIT- JEE Main 2016 Paper solution
IIT- JEE Main 2016 Paper solutionIIT- JEE Main 2016 Paper solution
IIT- JEE Main 2016 Paper solution
 
IIT - JEE Main 2016 Sample Paper -5
IIT - JEE Main 2016 Sample Paper -5IIT - JEE Main 2016 Sample Paper -5
IIT - JEE Main 2016 Sample Paper -5
 
IIT - JEE Main 2016 Sample Paper 3
IIT - JEE Main 2016 Sample Paper 3IIT - JEE Main 2016 Sample Paper 3
IIT - JEE Main 2016 Sample Paper 3
 
IIT - JEE Main 2016 Sample Paper -4
IIT - JEE Main 2016 Sample Paper -4IIT - JEE Main 2016 Sample Paper -4
IIT - JEE Main 2016 Sample Paper -4
 
IIT- JEE Main 2016 Sample Paper-2
IIT- JEE Main 2016 Sample Paper-2IIT- JEE Main 2016 Sample Paper-2
IIT- JEE Main 2016 Sample Paper-2
 
IIT- JEE Main 2016 Sample Paper-1
IIT- JEE Main 2016 Sample Paper-1IIT- JEE Main 2016 Sample Paper-1
IIT- JEE Main 2016 Sample Paper-1
 
Crash-Course for AIPMT & Other Medical Exams 2016(Essentials heart)
Crash-Course for AIPMT & Other Medical Exams 2016(Essentials heart)Crash-Course for AIPMT & Other Medical Exams 2016(Essentials heart)
Crash-Course for AIPMT & Other Medical Exams 2016(Essentials heart)
 
Crash-Course for AIPMT & Other Medical Exams 2016Target pmt (2)
Crash-Course for AIPMT & Other Medical Exams 2016Target pmt (2)Crash-Course for AIPMT & Other Medical Exams 2016Target pmt (2)
Crash-Course for AIPMT & Other Medical Exams 2016Target pmt (2)
 
Crash-Course for AIPMT & Other Medical Exams 2016 (Essentials cockroach)
Crash-Course for AIPMT & Other Medical Exams 2016 (Essentials cockroach)Crash-Course for AIPMT & Other Medical Exams 2016 (Essentials cockroach)
Crash-Course for AIPMT & Other Medical Exams 2016 (Essentials cockroach)
 
Class X SA-II MATHEMATICS SAMPLE PAPER 2016
Class X SA-II MATHEMATICS SAMPLE PAPER 2016Class X SA-II MATHEMATICS SAMPLE PAPER 2016
Class X SA-II MATHEMATICS SAMPLE PAPER 2016
 
Class X SA-II SCIENCE SAMPLE PAPER 2016
Class X SA-II SCIENCE SAMPLE PAPER 2016Class X SA-II SCIENCE SAMPLE PAPER 2016
Class X SA-II SCIENCE SAMPLE PAPER 2016
 
Class X SA-II SCIENCE SAMPLE PAPER 2016
Class X SA-II SCIENCE SAMPLE PAPER 2016Class X SA-II SCIENCE SAMPLE PAPER 2016
Class X SA-II SCIENCE SAMPLE PAPER 2016
 
I.S.C. Class XII MATHEMATICS Sample Papers 2016
I.S.C. Class XII MATHEMATICS Sample Papers 2016I.S.C. Class XII MATHEMATICS Sample Papers 2016
I.S.C. Class XII MATHEMATICS Sample Papers 2016
 
I.S.C. Class XII Sample Papers 2016
I.S.C. Class XII Sample Papers 2016I.S.C. Class XII Sample Papers 2016
I.S.C. Class XII Sample Papers 2016
 
I.S.C. Class XII Sample Papers 2016
I.S.C. Class XII Sample Papers 2016I.S.C. Class XII Sample Papers 2016
I.S.C. Class XII Sample Papers 2016
 
Crash Course For IIT-Main sample paper 2016
Crash Course For IIT-Main sample paper 2016Crash Course For IIT-Main sample paper 2016
Crash Course For IIT-Main sample paper 2016
 
SUMMATIVE ASSESSMENT-II MATHS SAMPLE TEST
SUMMATIVE ASSESSMENT-II MATHS SAMPLE TEST SUMMATIVE ASSESSMENT-II MATHS SAMPLE TEST
SUMMATIVE ASSESSMENT-II MATHS SAMPLE TEST
 
Prospectus FOR IIT-JEE, AIPMT, NTSE, OLYMPIAD
Prospectus FOR IIT-JEE, AIPMT, NTSE, OLYMPIADProspectus FOR IIT-JEE, AIPMT, NTSE, OLYMPIAD
Prospectus FOR IIT-JEE, AIPMT, NTSE, OLYMPIAD
 
SUMMATIVE ASSESSMENT-II MATHS SAMPLE TEST
SUMMATIVE ASSESSMENT-II MATHS SAMPLE TEST SUMMATIVE ASSESSMENT-II MATHS SAMPLE TEST
SUMMATIVE ASSESSMENT-II MATHS SAMPLE TEST
 
AIPMT-2016 SAMPLE TEST PAPER-14
AIPMT-2016 SAMPLE TEST PAPER-14AIPMT-2016 SAMPLE TEST PAPER-14
AIPMT-2016 SAMPLE TEST PAPER-14
 

Dernier

Advanced Views - Calendar View in Odoo 17
Advanced Views - Calendar View in Odoo 17Advanced Views - Calendar View in Odoo 17
Advanced Views - Calendar View in Odoo 17Celine George
 
JAPAN: ORGANISATION OF PMDA, PHARMACEUTICAL LAWS & REGULATIONS, TYPES OF REGI...
JAPAN: ORGANISATION OF PMDA, PHARMACEUTICAL LAWS & REGULATIONS, TYPES OF REGI...JAPAN: ORGANISATION OF PMDA, PHARMACEUTICAL LAWS & REGULATIONS, TYPES OF REGI...
JAPAN: ORGANISATION OF PMDA, PHARMACEUTICAL LAWS & REGULATIONS, TYPES OF REGI...anjaliyadav012327
 
social pharmacy d-pharm 1st year by Pragati K. Mahajan
social pharmacy d-pharm 1st year by Pragati K. Mahajansocial pharmacy d-pharm 1st year by Pragati K. Mahajan
social pharmacy d-pharm 1st year by Pragati K. Mahajanpragatimahajan3
 
microwave assisted reaction. General introduction
microwave assisted reaction. General introductionmicrowave assisted reaction. General introduction
microwave assisted reaction. General introductionMaksud Ahmed
 
Mastering the Unannounced Regulatory Inspection
Mastering the Unannounced Regulatory InspectionMastering the Unannounced Regulatory Inspection
Mastering the Unannounced Regulatory InspectionSafetyChain Software
 
Paris 2024 Olympic Geographies - an activity
Paris 2024 Olympic Geographies - an activityParis 2024 Olympic Geographies - an activity
Paris 2024 Olympic Geographies - an activityGeoBlogs
 
Sports & Fitness Value Added Course FY..
Sports & Fitness Value Added Course FY..Sports & Fitness Value Added Course FY..
Sports & Fitness Value Added Course FY..Disha Kariya
 
Organic Name Reactions for the students and aspirants of Chemistry12th.pptx
Organic Name Reactions  for the students and aspirants of Chemistry12th.pptxOrganic Name Reactions  for the students and aspirants of Chemistry12th.pptx
Organic Name Reactions for the students and aspirants of Chemistry12th.pptxVS Mahajan Coaching Centre
 
Arihant handbook biology for class 11 .pdf
Arihant handbook biology for class 11 .pdfArihant handbook biology for class 11 .pdf
Arihant handbook biology for class 11 .pdfchloefrazer622
 
A Critique of the Proposed National Education Policy Reform
A Critique of the Proposed National Education Policy ReformA Critique of the Proposed National Education Policy Reform
A Critique of the Proposed National Education Policy ReformChameera Dedduwage
 
Kisan Call Centre - To harness potential of ICT in Agriculture by answer farm...
Kisan Call Centre - To harness potential of ICT in Agriculture by answer farm...Kisan Call Centre - To harness potential of ICT in Agriculture by answer farm...
Kisan Call Centre - To harness potential of ICT in Agriculture by answer farm...Krashi Coaching
 
Activity 01 - Artificial Culture (1).pdf
Activity 01 - Artificial Culture (1).pdfActivity 01 - Artificial Culture (1).pdf
Activity 01 - Artificial Culture (1).pdfciinovamais
 
BASLIQ CURRENT LOOKBOOK LOOKBOOK(1) (1).pdf
BASLIQ CURRENT LOOKBOOK  LOOKBOOK(1) (1).pdfBASLIQ CURRENT LOOKBOOK  LOOKBOOK(1) (1).pdf
BASLIQ CURRENT LOOKBOOK LOOKBOOK(1) (1).pdfSoniaTolstoy
 
The basics of sentences session 2pptx copy.pptx
The basics of sentences session 2pptx copy.pptxThe basics of sentences session 2pptx copy.pptx
The basics of sentences session 2pptx copy.pptxheathfieldcps1
 
9548086042 for call girls in Indira Nagar with room service
9548086042  for call girls in Indira Nagar  with room service9548086042  for call girls in Indira Nagar  with room service
9548086042 for call girls in Indira Nagar with room servicediscovermytutordmt
 
The Most Excellent Way | 1 Corinthians 13
The Most Excellent Way | 1 Corinthians 13The Most Excellent Way | 1 Corinthians 13
The Most Excellent Way | 1 Corinthians 13Steve Thomason
 
Accessible design: Minimum effort, maximum impact
Accessible design: Minimum effort, maximum impactAccessible design: Minimum effort, maximum impact
Accessible design: Minimum effort, maximum impactdawncurless
 
Presentation by Andreas Schleicher Tackling the School Absenteeism Crisis 30 ...
Presentation by Andreas Schleicher Tackling the School Absenteeism Crisis 30 ...Presentation by Andreas Schleicher Tackling the School Absenteeism Crisis 30 ...
Presentation by Andreas Schleicher Tackling the School Absenteeism Crisis 30 ...EduSkills OECD
 
mini mental status format.docx
mini    mental       status     format.docxmini    mental       status     format.docx
mini mental status format.docxPoojaSen20
 

Dernier (20)

Advanced Views - Calendar View in Odoo 17
Advanced Views - Calendar View in Odoo 17Advanced Views - Calendar View in Odoo 17
Advanced Views - Calendar View in Odoo 17
 
JAPAN: ORGANISATION OF PMDA, PHARMACEUTICAL LAWS & REGULATIONS, TYPES OF REGI...
JAPAN: ORGANISATION OF PMDA, PHARMACEUTICAL LAWS & REGULATIONS, TYPES OF REGI...JAPAN: ORGANISATION OF PMDA, PHARMACEUTICAL LAWS & REGULATIONS, TYPES OF REGI...
JAPAN: ORGANISATION OF PMDA, PHARMACEUTICAL LAWS & REGULATIONS, TYPES OF REGI...
 
social pharmacy d-pharm 1st year by Pragati K. Mahajan
social pharmacy d-pharm 1st year by Pragati K. Mahajansocial pharmacy d-pharm 1st year by Pragati K. Mahajan
social pharmacy d-pharm 1st year by Pragati K. Mahajan
 
microwave assisted reaction. General introduction
microwave assisted reaction. General introductionmicrowave assisted reaction. General introduction
microwave assisted reaction. General introduction
 
Mattingly "AI & Prompt Design: Structured Data, Assistants, & RAG"
Mattingly "AI & Prompt Design: Structured Data, Assistants, & RAG"Mattingly "AI & Prompt Design: Structured Data, Assistants, & RAG"
Mattingly "AI & Prompt Design: Structured Data, Assistants, & RAG"
 
Mastering the Unannounced Regulatory Inspection
Mastering the Unannounced Regulatory InspectionMastering the Unannounced Regulatory Inspection
Mastering the Unannounced Regulatory Inspection
 
Paris 2024 Olympic Geographies - an activity
Paris 2024 Olympic Geographies - an activityParis 2024 Olympic Geographies - an activity
Paris 2024 Olympic Geographies - an activity
 
Sports & Fitness Value Added Course FY..
Sports & Fitness Value Added Course FY..Sports & Fitness Value Added Course FY..
Sports & Fitness Value Added Course FY..
 
Organic Name Reactions for the students and aspirants of Chemistry12th.pptx
Organic Name Reactions  for the students and aspirants of Chemistry12th.pptxOrganic Name Reactions  for the students and aspirants of Chemistry12th.pptx
Organic Name Reactions for the students and aspirants of Chemistry12th.pptx
 
Arihant handbook biology for class 11 .pdf
Arihant handbook biology for class 11 .pdfArihant handbook biology for class 11 .pdf
Arihant handbook biology for class 11 .pdf
 
A Critique of the Proposed National Education Policy Reform
A Critique of the Proposed National Education Policy ReformA Critique of the Proposed National Education Policy Reform
A Critique of the Proposed National Education Policy Reform
 
Kisan Call Centre - To harness potential of ICT in Agriculture by answer farm...
Kisan Call Centre - To harness potential of ICT in Agriculture by answer farm...Kisan Call Centre - To harness potential of ICT in Agriculture by answer farm...
Kisan Call Centre - To harness potential of ICT in Agriculture by answer farm...
 
Activity 01 - Artificial Culture (1).pdf
Activity 01 - Artificial Culture (1).pdfActivity 01 - Artificial Culture (1).pdf
Activity 01 - Artificial Culture (1).pdf
 
BASLIQ CURRENT LOOKBOOK LOOKBOOK(1) (1).pdf
BASLIQ CURRENT LOOKBOOK  LOOKBOOK(1) (1).pdfBASLIQ CURRENT LOOKBOOK  LOOKBOOK(1) (1).pdf
BASLIQ CURRENT LOOKBOOK LOOKBOOK(1) (1).pdf
 
The basics of sentences session 2pptx copy.pptx
The basics of sentences session 2pptx copy.pptxThe basics of sentences session 2pptx copy.pptx
The basics of sentences session 2pptx copy.pptx
 
9548086042 for call girls in Indira Nagar with room service
9548086042  for call girls in Indira Nagar  with room service9548086042  for call girls in Indira Nagar  with room service
9548086042 for call girls in Indira Nagar with room service
 
The Most Excellent Way | 1 Corinthians 13
The Most Excellent Way | 1 Corinthians 13The Most Excellent Way | 1 Corinthians 13
The Most Excellent Way | 1 Corinthians 13
 
Accessible design: Minimum effort, maximum impact
Accessible design: Minimum effort, maximum impactAccessible design: Minimum effort, maximum impact
Accessible design: Minimum effort, maximum impact
 
Presentation by Andreas Schleicher Tackling the School Absenteeism Crisis 30 ...
Presentation by Andreas Schleicher Tackling the School Absenteeism Crisis 30 ...Presentation by Andreas Schleicher Tackling the School Absenteeism Crisis 30 ...
Presentation by Andreas Schleicher Tackling the School Absenteeism Crisis 30 ...
 
mini mental status format.docx
mini    mental       status     format.docxmini    mental       status     format.docx
mini mental status format.docx
 

Permutation & combination

  • 1. 1.00 Introduction A branch of mathematics where we count number of objects or number of ways of doing a particular job without actually counting them is known as combinatorics and in this chapter we will with elementary combinatorics. For example, if in a room there are five rows of chairs and each row contains, seven hours, then without counting them we can say, total number of chairs is 35. We start this chapter principle of product or fundamental principle of counting.FUNDAMENTAL PRINCIPLE OF COUNTING 1.01 Fundamental Principal Of Counting MULTIPLICATION PRINCIPLE OF COUNTING If a job can be mm ways, and when it is done in any one of these ways another job be done in n, then both the jobs together can be done in mn ways. The rule can be extended to move number of jobs. Illustration 1. A college offers 7 courses in the morning and 5 in the evening. Find the possible number of choices with the student if he wants to study one course in the morning and one in the evening. Solution: The student has seven choices from the morning courses out of which he can select one course in 7 ways. For the evening course, he has 5 choices out of which he can select one in 5 ways. Hence the total number of ways in which he can make the choice of one course in the morning and one in the evening = 7 x 5 = 35. Illustration 2. A person wants to go from station A to station C via station B. There are three routes from A to B and four routes from B to C. In how many ways can he travel from A to C? Solution: A B in 3 ways B C in 4 ways A C in3x4 =12ways Remark: The rule of product is applicable only when the number of ways of doing each part is independent of each other i.e. corresponding to any method of doing the first part, the other part can be done by any method Illustration 3. How many (1) 5— digit (ii) 3 digit numbers can be formed by using 1, 2, 3, 4, 5 without repetition of digits. Solution: (i) Making a 5-digit number is equivalent to filling 5 places. Places: 1 2 3 4 5 Number of Choices: 5 4 3 2 1 The first place can be filled in 5 ways using anyone of the given digits.
  • 2. The second place can be filled in 4 ways using any of the remaining 4 digits. Similarly, we can fill the 3rd, 4th and 5th place. No. of ways of filling all the five places = 5 x 4 x 3 x 2 x 1 = 120 120 5-digit numbers can be formed. (ii) Making a 3-digit number is equivalent to filling 3 places. Places: l 1 2 3 Number of Choices: 5 4 3 Number of ways of filling all the three places = 5 x 4 x 3 = 60 Hence the total possible 3-digit numbers = 60. Illustration 4. There are 10 steamers plying between Liverpool and Dublin; in how many ways can a man go from Liverpool to Dublin and return by a different steamer? Solution: There are ten ways of making the first passage; and with each of these there is a choice of nine ways of returning (since the man is not to come hack by the same steamer); hence the number of ways of making the two journeys is 10 x 9, or 90. This principle may easily be extended to the case in which there are more than two operations each of which can be performed in a given number of ways. Addition Principle : If one experiment has n possible outcomes and another has m possible outcomes, then there are (m + n) possible outcomes when exactly one of these experiments is performed. In other words, if a job can be done by n methods and by using the first method, can be done in a1 ways or by second method in 2 ways and so on . . . by the nth method in a ways, then the number of ways to get the job done is (a1 a .......... an ) . Illustration 5. A train is going from Cambridge to London stops at nine intermediate stations. Six persons enter the train during the journey with six different tickets. How many different sets of tickets they have had? Solution: For S1 ,9 different tickets available, one for each of the remaining 9 stations; similarly at S 2 . 8 different tickets available; and so on. Hence, it is clear, that total number of different tickets =9+8+7+6+5+4+3+2+1=45 Hence, the six different tickets must be any six of these 45; and there evidently as mar different sets of 6 tickets as there are combinations of 45 things taken 6 at a time. Illustration 6. A college offers 7 courses in the morning and 5 in the evening. Find the number J of ways a student can select exactly one course, either in the morning or in the evening. Solution: The student has seven choices from the morning courses out of which he can select one course in 7 ways. For the evening course, he has 5 choices out of which he can select one course in 5 ways.
  • 3. Hence he has total number of 7 + 5 = 12 choices. Illustration 7. A person wants to leave station B. There are three routes from station B o A and J four routes from B to C. In how many ways can he leave the station B. Solution: B A in 3 ways B C in 4 ways He can leave station B in 3 + 4 = 7 ways. 1.02 Permutations (Arrangement Of Objects) The number of permutations of n objects, taken r at a time, is the total number of arrangements of r objects, selected from n objects where the order of the arrangement is important. Without repetition: (a) Arranging n objects, taken r at a time is equivalent to filling r places from n things. r-places - 1 2 3 4 r No. of choices- n n-1 n-2 n-3 n-(r-1) The number of ways of arranging = the number of ways of filling r places =n (n—1) (n—2) (n—r+1) n(n 1)(n 2)...................(n r 1((n r )!) n! n = Pr (n r )! (n r )! n (b) The number of arrangements of n different objects taken all at a time Pn n! With repetition: (a) The number of permutations (arrangements) of n different objects, taken r at a time, when each object may occur once, twice, thrice .... up to r times in any arrangement = The number of ways of filling r places where each place can be filled by any one of n objects. r-places - 1 2 3 4 r No. of choices- n n n n n r The number of permutations = the number of ways of filling r places = ( n ) (b) The number of arrangements that can be formed using n objects out of which p are identical (and of one kind), q n! are identical (and of another kind), r are identical (and of another kind) and the rest are distinct is p !q !r ! Illustration 8. How many 7- letter words can be formed using the letters of the words? (a) BELFAST (b) ALABAMA
  • 4. Solution: (a) BELFAST has all different letters. Hence the number of words= 7 P7 = 7! = 5040 (b) ALABAMA has 4 A’s but the rest are all different. Hence the number of words that can be formed is 7!/4!= 7 x 6 x 5 = 210. Illustration 9. a. How many anagrams can be made by using the letters of the word HINDUSTAN. b. How many of these anagrams begin and end with a vowel. c. ln how many of these anagrams, all the vowels come together. d. In how many of these anagrams, none of the vowels come together. e. In how many of these anagrams, do the vowels and the consonants occupy the same relative positions as in HINDUSTAN. Solution: (a) The total number of anagrams = Arrangements of nine letters taken all at a time = 9! /2! = 181440. (b) We have 3 vowels and 6 consonants, in which 2 consonants are alike. The first place can be filled in 3 ways and the last in 2 ways. The rest of the places can be filled in 7! / 2! ways. Hence the total number of anagrams 3 x 2 x 7! / 2! = 15120 (c) Assume the vowels (I, U, A) as a single letter. The letters (IUA) H, D, S, T, N, N can be arranged in 7! / 2! ways. Also IUA can be arranged among themselves in 3! = 6 ways. Hence the total number of anagrams = 7! / 2! x 6 = 15120 (d) Let us divide the task into two parts. In the first, we arrange the 6 consonants as shown below in 6! / 2! ways. x C x C x C x C x C x C x (C stands for consonants and x stands for blank spaces in between them) Now 3 vowels can be placed in 7 places (in between the consonants) in 7 P3 7! — =210 ways. 4! Hence the total number of anagrams =6! / 2! x 210 = 75600. (e) In this case, the vowels can be arranged among themselves in 3! = 6ways. Also, the consonants can be arranged among themselves in 6! / 2! ways. Hence the total number of anagrams 6! / 2! x 6= 2160. Illustration 10. How many 3 digit numbers can be formed using the digits 0, 1, 2,3,4,5 so that (a) digits may not be repeated (b) digits may be repeated. Solution: (a) Let the 3-digit number be XYZ Position (X) can be filled by 1, 2,3,4,5 but not 0. So it can be filled in 5 ways.
  • 5. Position (Y) can be filled in 5 ways again. (Since 0 can be placed in this position). Position (Z) can be filled in 4 ways. Hence by the fundamental principle of counting, total number of ways is 5 x 5 x 4 = l00ways. (b) Let the 3 digit number be XYZ Position (X) can be filled in 5 ways Position (Y) can be filled in 6ways. Position (Z) can be filled in 6 ways. Hence by the fundamental principle of counting, total number of ways is 5 x 6 x 6 = 180 1.03 Important Results Number of permutations under certain conditions: • Number of permutations of n different things, taken r at a time, when particular thing s to be always included in each arrangement is r n 1 Pr 1 • Number of permutations of n different things, taken r at a time, when a particular thin; is never taken in any arrangement is n 1Pr . • Number of permutations of n different things, taken r at a time, when m particular things are never taken in any n m arrangement is Pr Number of permutations of n different things, taken all at a time, when m specific things always come together is (m!) (n — m + 1)!. • Number of permutations of n different things, taken all at a time, when m specified things never come together is n! — m! (n — m + 1)! Illustration 11. There are m men and n monkeys (n > m). if a man have any number of monkeys .in how many ways may every monkeys have a master? Solution: The first monkey can select his master by m ways and after that the second monkey can select his master again by m ways, so can the third. And so on, n hence all monkeys can select master = m x m x m…up to n times = m ways Illustration 12. Find the number of ways in which we can arrange four letters of the word MATHEMAT. Solution: Letters of the word MATHEMATICS are (M, M), (A, A), (T, T), H, E, I, C and S, making eight distinct letters. We can choose four out of them in 8C4 70 ways, and arrange each of these sets of four in 4! = 24 ways, yielding (70) (24) = 1680 arrangements. Second, we can choose one pair from
  • 6. among the three identical letter pairs, and two distinct letters out of the remaining seven in 3 7 C1 C2 3 7 6 /2 63 ways. The letters so obtained can be arranged in 4! /2! = 12 ways, so the number of arrangements in this case is (63) (12) = 756. Finally, we can choose two pairs out of the three identical letter pairs. This can be done in 3C2 ways and the letters obtained can be arranged in 4! /2! 2! = 6 ways, so that the number of arrangements in this last case is (3) (6) = 18. Hence the total number of arrangements is 1680 + 756 + 18 = 2454. 1.04 Cicular Permutations In the event of the given n things arranged in a circular or even elliptical permutation — and in this case the first and the last thing in the arrangement are indistinguishable — the number of :―permutations is (n—1)!. For example is 20 persons are circularly arranged, the number of arrangements is 19! If positions on a circular arrangement are numbered, then can it be treated as linear arrangement? however, in the case of circular permutations wherein clockwise and anticlockwise orders need not be differentiated — as in the case of differently coloured beads or different flowers made in to a garland, the number of permutations is n 1! where the number of beads or flowers is taken as n. In details the concept is explained as follows: 2 ARANGEMENTS AROUND A CIRCULAR TABLE Consider five persons A, B, C, D, E be seated on the circumference of a circular table in Order which has no head now, shifting A, B, C, D, E one position in anticlockwise direction we will get arrangements as shown in following figure: We observe that arrangements in all figures are different. Thus, the number of circular permutations of n different things taken all at a time is (n — 1)!, if clockwise and anticlockwise orders are taken as different. Illustration 13. 20 persons were invited to a party. In how many ways can they and the host be seated at a circular table? In how many of these ways will two particular persons be seated on either side of the host? Solution: 1st part: Total persons on the circular table = 20 guest + 1 host = 21 they can be seated in (21 — 1)! = 20! ways.
  • 7. 2nd part: After fixing the places of three persons (1 host + 2 persons). Treating (1 host + 2 person) = 1 unit, so we have now ((remaining 18 persons + 1 unit) = 19) and the number of arrangement will be (19 — 1)! = 18! also these two particular person can be seated on either side of the host in 2! ways. Hence, the number of ways of seating 21 persons on the circular table such that two particular persons be seated on either side of the host= 18! x 2! =2x18! 1.05 Arrangements Of Beads Or Flowers (All Different) Around A Circular Necklace Or Garland Consider five beads A, B, C, D, E in a necklace or five flowers A, B, C, D, E in a garland etc. If the necklace or garland on the left is turned over we obtain the arrangement on the right. i.e., anticlockwise and clockwise order of arrangement is not different we will get arrangements as follows: We see that arrangements in above figures are not different. 1 Then the number of circular permutations of n different things taken all at a time is (n 1)! if clockwise and 2 anticlockwise orders are taken as not different. Illustration 14. Consider 21 different pearls on a necklace. How many ways can the pearls be placed in on this necklace such that 3 specific pearls always remain together? Solution: After fixing the places of three pearls. Treating 3 specific pearls = 1 units, so we have now 18 pearls + 1 unit = 19 and the number of arrangement will be (19 — 1)! = 18! also, the number of ways of 3 pearls can be arranged between themselves is 3! 6. Since, there is no distinction between the 1 clockwise and anticlockwise arrangements. So, the required number of arrangements = 18!. 6 = 3 2 (18!).
  • 8. NUMBER OF CIRCULAR PERMUTATIONS OF n DIFFERENT THINGS TAKEN r AT A TIME CASE I - If clockwise and anticlockwise orders are taken as different, then the required number of circular n Pr n permutations= or Cr (r 1)! r Illustration 15. In how many ways can 24 persons be seated round a table, if there are 13 seats? Solution: In case of circular table the clockwise and anticlockwise order are different, then the required number 24 P13 24! of circular permutations = 13 13 11! Case II - If clockwise and anticlockwise orders are taken as not different, then the required number of circular n n Pr Cr (r 1)! permutations = or 2r 2 Illustration 16. How many necklace of 12 beads each can be made from 18 beads of various colours? Solution: In the case of necklace there is not distinction t between the clockwise and anticlockwise arrangements, then the required number of circular permutations 18 P12 18! 18 17 16 15 14 13! 119 13! 2 12 24 6! 6 5 4 3 2 1 24 2 Illustration 17. In how many ways 10 boys and 5 girls can sit around a circular table so that no two girls sit together. Solution: 10 boys can be seated in a circle in 9! ways. There are 10 spaces in between the boys, which can be 10 10 occupied by 5 girls in P5 ways. Hence total numbers of ways are 9! P5 .or 9! 10 C5 (5!) . Illustration 18. If n distinct objects are arranged in a circle, show that the number of ways of selecting three of these n(n 4)(n 5) n things so that no two of them is next to each other is 6 Solution: Let the n things be x1 , x2 ,..........................xn . The first choice may be one of these n things; and, this is done in n C1 ways. Suppose x1 is the one chosen; the next two may be chosen — excluding x1 and, the two next to x1 , n 3 n 3 namely x 2 , x n —from the remaining (n —3) in C 2 ways. Of these C 2 there are (n — 4) selections when the second two chosen are next to each other, like x3 x4 .x4 x5 ,..........................xn 2 xn 1 . the number of ways of selecting the second two after x1 is chosen, so that the two are not next to each other is
  • 9. n 3 (n 3)(n 4) (n 4)(n 5) C2 (n 4) (n 4) 1.2 2 The two objects can be relatively interchanged in 2 ways. Further the order of the choice of the three is not to be considered. Hence the number of ways of choice of the three is n(n 4)(n 5) 2! n(n 4)(n 5) 2 3! 6 Illustration 19. 2n persons are to be seated n on each side of a long table. r(<n) particular persons desire to sit on one side; and s(<n) other persons desire to sit on the other side. In how many ways can the persons be seated? Solution: For the side where r persons desire to sit, we need (n — r) more persons. This (n —r) may be chosen from (2n — r — s) in (2 n r s ) Cn r ways. Automatically the remaining (n — s) person go to the other (2 n r s ) side where already there are s desirous of seating. Thus there are Cn r ways of distributing n persons for each side providing for the restriction of on one side and s on the other side. n persons on each side can be permuted in n seats in n! ways. The number of ways of seating the 2n persons, n on each side, is therefore (2 n r s ) 2 Cn r n! S 1.06 Combinations Each of different grouping or selections that can be made by some or all of a number of given things without considering the order in which things are placed in each group, is called COMBINATIONS Can combinations be considered for both non-identical and identical objects? COUNTING FORMULAE FOR COMBINATIONS The number of combinations (selections or groupings) that can be formed from n different objects taken r at a time is denoted by n C r and its value is equal to n n n! Pr ! Cr 0 r n as n Cr n r !r ! r! as in a permutation the arrangement of r selected objects out of n, is done in r! ways and in combination arrangement in a group is not considered. In particular  n C0 n Cn 1 i.e. there is only one way to select none or to select all objects out of n distinct objects.
  • 10. n C1 n There are n ways to select one thing out of n distinct things. n n  n Cr n Cn r therefore Cx Cy x y or x y n n  If n is odd then the greatest value of n C r is Cn 1 or n Cn 1 2 2 n  If n is even then the greatest value of n C r is Cn 2 If the number of ways of selection of r objects out of 2n objects is maximum then, What can be the possible value of r? Illustration 20. Six X’s have to be placed in spaces on the adjoining Figure so that each row contains at least one X. In how many different ways this can be done? Solution: If there be no restriction on the placing of the) (s, the number of ways is 8 C6 8 C2 28 these there are two ways in which the X; s can be placed; one, with the first row empty and the other with the third row empty. These two cases only do not satisfy the condition. So the number of ways = 28— 2 = 26. IMPORTANT RESULTS OF COMBINATIONS (SELECTIONS)  The number of ways in which r objects can be selected from n distinct objects if a particular object is always n 1 included is Cr 1 .  The number of ways in which r objects can be selected from n distinct objects if a particular object is always n 1 excluded is Cr .  The number of ways in which r objects can be selected from n distinct objects if m particular objects are always included is n n m Cr m .  The number of ways in which r objects can be selected from n distinct objects if m particular objects are always excluded is n n m Cr . Illustration 21. A lady desires to give a dinner party for 8 guests. In how many ways can the lady select guests for the dinner from her 12 friends, if two of the guests will not attend the party together? Solution: The following three methods of approach are indicated. (i) Number of ways of forming the party
  • 11. 12 10 10 = C8 C6 since C6 is the number of ways of making up the party with both the specified guests included. =495—210=285 (OR) (ii) Number of ways of forming the party = Number of ways of forming without both of them + Number of ways of forming with one of them and without the other 10 = C8 2.10 C7 45 240 285 (OR) (iii) Split the number of ways of forming the party = those with one of the two (say A) + those without A 10 = C7 2.11 C8 120 165 285 Note- The number of ways in which r objects can be selected from n objects if m particular r r n m n m objects are identical is Cr or Cr according as r m or r m r 0 r r m Illustration 22. A bag contains 23 balls in which 7 are identical. Then find the number of ways of selecting 12 balls from bag. Solution: Here n=23, p=7, r=12(r>p) 12 16 Hence, required number of selections Cr r 5 16 16 16 16 16 16 16 16 C5 C6 C7 C8 C9 C10 C11 C12 16 16 16 16 16 16 16 16 C5 C6 C7 C8 C9 C10 C11 C12 17 C6 17 C8 17 C10 17 C12 n Cr n Cr 1 n 1 Cr 17 C11 17 C9 17 C10 17 C12 n Cr n Cn r 17 17 17 17 C11 C12 C9 C10 18 18 18 18 C12 C10 C6 C8 1.07 Selection From Distinct / Identical Objects SELECTION FROM DISTINCT OBJECTS
  • 12. The number of ways (or combinations) of selection from n distinct objects, taken at least one of them is n n n n C1 C2 C3 .......................................... Cn 2n 1 Logically it can be explained in two ways, as one can be selected in n C1 ways, two in n C 2 ways and so on and by addition principle of counting the total number of ways of doing either of the job is n n n n C1 C2 C3 .......... Cn Also, for every object, there are two choices, either selection or non-selection. Hence total choices are 2 n . But this also includes the case when none of them is selected. Therefore the number of selections, when at least one is selected = 2n 1 It is allowed to select at most 5 things out of 11 different things. In how many ways at least one of them can be selected? Illustration 23. Given five different green dyes, four different blue dyes and three different red dyes, how many combination of dyes can be chosen taking at least one green, one blue dye? Solution: Any one dye of a particular colour can be either chosen or not; and, thus there are 2 ways in which each one may be dealt with. Number of ways of selection so that at least one green dye is included 25 1 31 (1 is subtracted to correspond to the case when none of the green dyes is chosen.) A similar argument may be advanced in respect of other two colours also. Number of combinations = 25 1 24 1 23 31 15 8 3720 SEL ECTION FROM IDENTICAL OBJECTS 1. The number of selections of r r n objects out of n identical objects is 1. 2. The number of ways of selections of at least one object out of n identical object is n. 3. The number of ways of selections of at least one out of a1 , a2 , a3 ...........an objects, where a1 are alike of one kind, a 2 are alike of second kind, and so on a n are alike of the nth kind, is a1 1 a2 1 ................... an 1 1 4. The number of ways of selections of at least one out of a1 a2 a3 ................... an k objects, where a1 are alike of one kind a are alike of nth kind and k are distinct is a1 1 a2 1 ................... an 1 2k 1
  • 13. Illustration 24. Find the number of combinations that can be formed with 5 oranges, 4 mangoes and 3 bananas when it is essential to take (i) at least one fruit (ii) one fruit of each kind. Solution: Here 5 oranges are alike of one kind, 4 mangoes are alike of second kind and 3 bananas are alike of third kind (i) The required number of combinations (when at least one fruit) = (5 + 1) (4 + 1) (3 + 1)2°—i = 120—1 =119 (ii) The required number of combinations (when one fruit of each kind) 5 4 3 C1 C1 C1 5 4 3 60 1.08 Divisors Of A Given Natural Number Let n N and n P 1 .P2 2 .P3 3 .....................Pk k , where P , P2 , P3 ..............Pk are different prime numbers and 1 1 1 , 2 , 3 .............. k are natural numbers then:  the total number of divisors of N including 1 and n is 1 1, 2 2 , 3 3 .............. k 1  the total number of divisors of n excluding 1 and n is 1 1, 2 2 , 3 3 .............. k 1 2  the total number of divisors of n excluding exactly one out of 1 or n is 1 1, 2 2 , 3 3 .............. k 1 1 1  the sum of these divisors is 1 1, 2 1 ,.............. k 1 2 = P01 , P1 , P 2 1 1 .......P 1 1 P02 , P21 , P22 .......P2 2 .......... P0 k , Pk1 , Pk 2 .......Pk k (Use sum of G.P. in each bracket)  the number of ways in which n can be resolved as a product of two factors is if n is not a perfect square 1 1 1, 2 1 ,.............. k 1 1 if n is a perfect square 2  the number of ways in which is composite number n can be resolved into two factors which are relatively prime k 1 (or co prime) to each other is equal to 2 where k is the number of different factors (or different primes) in n. Illustration 25. If n = 10800then find the (a) total number of divisors of n (b) the number of even divisors
  • 14. (c) the number of divisors of the form 4m + 2 (d) the number of divisors which are multiples of 15 4 Solution: n= 10800 = 2 33 52 a Any divisor of n will be of the form 2 3b 5c where 0 a 4,0 b 3,0 c 2. For any distinct choices of a, b and c, we get a divisor of n (a) total number of divisors = (4 + 1) (3 + 1) (2 + 1) = 60 (b) for a divisor to be even, a should be at least one. So total number of even divisors = 4(3 + 1) (2 + 1) = 48. (c) 4m + 2 = 2(2m + 1). In any divisor of the form 4m + 2, a should be exactly 1. So number of divisors of the form 4m + 2 = 1(3 + 1) (2 + 1) = 12. (d) A divisor of n will be a multiple of 15 if b is at least one and c is at least one. So number of such divisors = (4+1) x 3 x 2 = 30. (8) Division into groups: (i) The number of ways in which (m + n) different things can be divided into two groups which contain m and n things respectively is m n (m n)! Cm nCn ;m n m!n ! Corollary : If m = n, then the groups are equal size. Division of these groups can be given by two types. Type I : If order of group is not important: The number of ways in which 2n different things can be divided equally into two groups is (2n)! 2!( n !) 2 Type II: If order of group is important: The number of ways in which 2n different things can be divided equally into two distinct groups is (2n)! 2n ! 2 2! 2!(n !) ( n !) 2 (ii) The number of ways in which (m + n +p) different things can be divided into three groups which contain m , n and p things respectively is m n p (m n p )! Cm n p C n p C p ;m n p m !n ! p ! Corollary : If m = n = p, then the groups are equal size. Division of these groups can he given by two types. Type .I : If order of group is not important: The number of ways in which 3p different things can be divided equally (3 p )! into three groups is 3!( p !)3
  • 15. Type II : If order of group is important : The number of ways in which 3p different things can be divided equally into three distinct groups is (3 p)! 3 p! 3 3! 3!( p !) ( p !)3 Note: (1) If order of group is not important: The number of ways in which mn different things can be divided equally into m mn ! groups is ( n !) m m ! (2) If order of group is important : The number of ways in which mn different things can be divided equally into m mn ! (mn)! distinct groups is m m! (n !) m ! (n !) m Illustration 26. In how many ways can a pack of 52 cards be equally among 4 players in order? . ‘ Solution. Here order ot group is important, then. the numbers of ways in which 52 different cards can be 52! (52)! divided equally into 4 players is 4! 4!(13!)4 (13!) 4 Alternative method : Each player will get 13 cards. Now first player can be given 13 cards out of 52 cards in 52C13 ways. Second player can be given 13 cards out of remaining 39 cards (i.e. 52— 13 = 39) in 39C13 ways. Third player can be given 13 cards out of remaining 26 cards (i.e., 39 — 13 = 26) in 26C13 ways and fourth player can be given 13 cards out of remaining 13 cards (i.e. 26— 13 = 13) in 13C13 ways. Hence required number of ways 52 39 26 13 C13 C13 C13 C13 52! 39! 26! 1 13! 39! 13! 26! 13! 13! 52! 4 13! Illustration 27. In how many ways can a pack of 52 cards be formed into 4 groups of 13 cards each ? Solution. Here order of group is not important, then the number of ways in which 52 different cards can be divided equally into 4 groups is 52! 4!(13!) 4 Alternative Method: Each group will get 13 cards. Now first group can be given 13 cards out of 52 cards in C13 ways. Second group can be given 13 cards out of remaining 39 cards (i.e. 52— 13 = 39) in 39C13 ways. Third group can be given 13 cards out of remaining 26 cards (i.e., 39 — 13 = 26) in 26C13 ways and fourth group can be given 13 cards out of remaining 13
  • 16. cards (i.e.,26 — 13 = 13) in 13C13 ways. But the all (four) groups can be interchanged in 4 ! ways. Hence the required number of ways 52 39 126 13 C13 C13 C13 C13 4! 52! 39! 26! 1 1 13! 39! 13! 26! 13! 13! 4! 52! 4 13! 4! Illustration28. In how many ways can a pack of 52 cards be divided in 4 sets, three of them having 17 cards each and fourth just card? (52)! Solution. First we divide 52 cards into two sets which contains 1 and 51 cards respectively is 1!51! Now 51 cards can be divided equally in three’ sets each contains 17 cards (Here order of sets is not (51)! important) in ways. 3!(17!)3 Hence the required number of ways (52)! (51)! 1!51! 3!(17!)3 (52)! (52)! 1!3!(17!)3 (17!)3 3! Alternative Method : First set can be given 17 cards out of 52 cards in 52C17. Second set can be given 17 cards out of remaining 35 cards (i.e. 52 — 17 = 35) in 35C17. Third set can be given 17 cards out of remaining 18 cards (i.e., 35 — 17 = 18) in 18C17 and fourth set can be given 1 card out of 1 card in 1C1. But the first three sets can be interchanged in 3 ! ways. Hence the total number of ways for the required distribution 52 35 1 18 1 C17 C17 C17 C1 3! 52 35! 18! 1 1 17! 35! 17! 18! 17! 1! 3! (52)! (17!)3 3! Illustration29. In how many ways can 12 balls be divided between 2 boys, one receiving 5 and the other 7 balls? Also in how many ways can these 12 balls be divided into groups of 5, 4 and 3 balls respectively ? Solution. I Part: Here order is important, then the number of ways in which 12 different balls can be divided between two boys which contains 5 and 7 balls respectively, is (12)! 2! 1584 5!7!
  • 17. Alternative Method: First boy can be given 5 balls out of 12 balls in 12C5. Second boy can be given 7 balls out of 7 balls (i.e. 12 — 5 = 7) but there order is important (boys interchange by (2 types) then required no. of ways 12 7 C5 C7 2! 12! 1 2! 5! 7! 12! 2 5! 7! 12. 11. 10. 9. 8. 7! .2 5. 4. 3. 2. 1. 7! 1584 II part : Here order is not important then the number of ways in which 12 different balls can be divided into three groups of 5, 4 and 3 balls respectively, is (12)! 27720 5!4!3! Alternative Method: First group can be given 5 balls out of 12 balls in C5 ways. Second group can be given 4 balls out of remaining 7 balls (12 — 5 = 7) in 7C4 and 3 balls can be given out of remaining 3 balls in 3C3. Hence the required number of ways (Here order of groups are not important) 12 7 3 C5 C4 C3 12! 7! 1 5! 7! 4! 3! 12! 5! 4! 3! (9) Arrangement in Groups: (i) The number of ways in which n different things can be arranged into r different groups is n r 1 Pn or n !n 1 Cr 1 according as blank group are or are not admissible. Illustration 30. In how many ways 5 different. balls can be arranged into 3 different boxes so that no box remains empty? Solution. The required number of ways = 5!.5 1 C3 1 5!.4 C2 720 Alternative Method : Each box must contain a least one ball since no box remains empty. Boxes can have balls in the following systems
  • 18. 3! All 5 balls can be arranged by 5 ! ways and boxes can be arranged in each system by 2! 3! 3! Hence required number of ways 5! 5! 720 2! 2! (ii) The number of ways in which n different things can be distributed into r different groups is rn r C1 (r 1) n r C2 ( r 2) n .................... ( 1) r 1r Cr 1 x coefficient of xn in n !(e 1) r Here blank groups are not allowed. pr Note Coefficient of xr in e px r! Illustration 31. In how many ways 5 different balls can be distributed into 3 boxes so that no box remains empty? Solution. The required number of ways 5 5 5 35 – 3 C1 3 – 1 3 C2 3 – 2 – 3 C3 3 – 3 243 – 96 3 – 0 150 OR Coefficient of x in 5! (e x – 1) 3 5 coeffi of x 5 in 5! e3 x – 3e 2 x 3e x – 1 35 25 1 5! –3 3 5! 5 5! 35 – 3.25 3 243 – 96 3 150 Alternative Method : Each box must contain at least one ball since no. box remains empty. Boxes can have balls in the following systems The number of ways to distribute the balls in I system = 5 C1 4 C1 3 C3 The total number of ways to distribute 1, 1, 3 balls to the boxes 5 4 3 3! = C1 C1 C3 2! 5 4 1 3 60 and the number of ways to distribute the balls in II system = 5 C1 4 C2 2 C2
  • 19. The total number of ways to distribute 1, 2, 2 balls to the boxes 5 4 2 3! = C1 C2 C2 2! 5 6 1 3 90 The required number of ways = 60 + 90 = 150 (iii) The number of ways in which n identical things can be distributed into r different groups is n r 1 n 1 Cr 1 or Cr 1 according as blank groups are or are not admissible. Illustration 32. In how many ways 5 identical balls can be distributed into 3 different boxes so that no box remains empty? 5 1 Solution. The required number of ways = C3 1 = 4C2 = 6 Alternative Method: Each box must contain at least one ball since no box remains empty. Boxes can have balls in the following systems Here balls are identical but boxes are different the number of combinations will be I in each systems. Required number of ways 3! 3! =1 1 6 2! 2! Illustration 33. Four boys picked up 30 mangoes. in how many ways can they divide them if all mangoes be identical? Solution. Clearly, 30 mangoes can be distributed among 4 boys such that each boy can receive any number of mangoes. Hence total number of ways 30 4 1 C4 1 = 33C3 =5456 Illustration 34. Find the positive number of solutions of x + y + z + w = 20 under the following conditions: (i) zero values of x, y , z, w are included. (ii) zero values are excluded. Solution. (i) Since x + y + z + w = 20 Here x 0, y 0, z 0, w 0 The number of solutions of the given equation in this case is same as the number of ways of distributing 20 things among 4 different groups. Hence total number of solutions = 20 4 1 C4 1 23 C3 1771 (ii) Since x+y+z+w = 20
  • 20. Here x 1, y 1, z 1, w 1 or x –1 0, y –1 0, z –1 0, w –1 0 Let x1 x –1 x x1 1 y1 y –1 y y1 1 z1 z –1 z z1 1 w1 w –1 w w1 1 Then from (1) x1 1 y1 1 z1 1 w1 1 20 x1 y1 z1 w1 16 and x1 0, y1 0, z1 0, w1 0 16 4–1 Hence total number of solutions C4–1 19 19. 18. 17 C3 1. 2. 3 57. 17 969. Alternative Method : Part (ii ) :  x y z w 20 x 1, y 1, z 1, w 1 20–1 19 Hence total no. of solutions C4–1 C3 969. Illustration 35. How many integral solutions are there to x + y + z + t = 29, when x 1,y l, z 3andt 0? Solution. Since x + y + z + t = 29 and x , y, z, t are integers x 1, y 2, z 3, t 0 x – 1 0, y – 2 0, z – 3 0, t 0 let x1 x – 1, x2 y – 2, x3 z –3 or x x1 1, y x2 2, z x3 3 andthen x1 0, x2 0, x3 0, t 0 From (1), x1 1 x2 2 x3 3 t 29 x1 x2 x3 t 23 23 4–1 Hence total number of solutions C4–1 26 26. 25. 24 C3 2600 1. 2. 3 Illustration 36. How many integral solutions are there to the system ‘of equations x1 +x2 +x3 +x4+x5 = 20 and x1 +x2 = 15 when xk 0? Solution. We have x1 +x2 +x3 +x4+x5 = 20 ………………(i) and x1 + x2 = 15 ………………(ii) then from (i) and (ii) we get two equation
  • 21. x3 x4 x5 5 x1 x2 15 and given x1 0, x2 0, x3 0, x4 0 and x5 0 5 3–1 Then number of solutions of equation (iii) C3–1 7 7. 6 C2 21 1. 2 and number of solutions of equation (iv) = 15 + 2— 1C2-1 = 16C1 = 16 Hence total number of solutions of the given system of equations = 21 x 16 =336. Illustration 37. Find the number of non-negative integral solutions of 3x + y + z = 24. Solution. We have let x = k 3x + y + z = 24 and given x 0, y 0, z 0 y+z = 24—3k Here 24 24 – 3k 0 Hence 0 k 8 The total number of integral solutions of (i )is 24–3 k 2–1 25–3 k C2–1 C1 25 – 3k Hence the total number of solutions of the original equation 8 8 8 25 – 3k 25 1– 3 k k 0 k 0 k 0 8.9 25. 9 – 3. 225 – 108 117 2 (iv) The number of ways in which n identical things can be distributed into r groups so that no group contains less than l things and more than in things (l < m) is ii. 1 1+1 1+2 in r l r coefficient of xn in the expansion of x xl 1 xl 2 ... xm lr l 1 r r or coefficient of xn in the expansion of x 1- x m 1 x Illustration 38. In how many ways can three persons, each throwing a single dia once, make a sum of 15? Solution. Numbers on the faces of the dia are 1, 2, 3, 4, 5, 6 (least number 1, greatest number 6)
  • 22. 3 Required number of ways coefficient of x15 in x1 x2 x3 x4 x5 x6 3 coeffi. of x15 in x 3 1 x x 2 x3 x4 x 3 coeffi. of x12 in 1 x x 2 x3 x4 x5 3 –3 coeffi. of x12 in 1 – x 6 1– x coeffi. of x12 in 1 – 3 x 6 3 x12 1 3 x 6 x 2 .... 28 x 6 ... 91x12 ... 91 – 84 3 94 – 84 10 Illustration 39. In how many ways in which an examiner can assign 30 marks to 8 questions, giving not less than 2 marks to any question. Solution. If examiner given marks any seven question 2 (each) marks then marks on remaining questions given by examiner = — 7 x 2+ 30 = 16 If x are the marks assigned to ith question, then x1 + x2 + x3 + ... + x8 = 30 and 2 xi l6fori =1,2,3 8. Therefore the required number of ways 30 2 3 16 8 coefficient of in ... 30 16 14 8 coeffi. of in 1 .... 14 14 8 coeffi. of in 1 ... 8 14 1– 15 coeffi. of in 1– –8 coeffi. of 14 in 1 – 14 8 9 2 21 14 coeffi. of in 1 C1 C2 ... C13 .... 21 21 C14 C7 Note: Coefficient Of xr in the expansion of(l —x)-n is n + r - 1Cr (v) If a group has n things in which p are identical, then the number of ways of selecting r things from a group is r r n– p n– p Cr or Cr according as r p or r p. r 0 r r– p
  • 23. Illustration 40. A bag has contains 23 balls in which 7 are identical. Then find the number of ways of selecting 12 balls from bag. Solution. Here n = 23,p=7,r=12 (r > p) 12 16 Required number of selections Cr r 5 16 16 16 16 16 16 16 16 C5 C6 C7 C8 C9 C10 C11 C12 16 16 16 16 16 16 16 16 C5 C6 C7 C8 C9 C10 C11 C12 17 C6 17 C8 17 C10 17 C12 n Cr n Cr –1 n 2 Cr 17 C11 17 C9 17 C10 17 C12 n Cr n Cn – r 17 17 17 17 C11 C12 C9 C10 18 18 C12 C10 18 18 C6 C8 Deragements If. n things are arranged in a row, the number of ways in which they can be deranged so that no one of them occupies its original place is 1 1 1 1 1 n! 1 ....................... ( 1) n 1! 2! 3! 4! n! No object goes to its scheduled place. Remark: If r things goes to wrong place out of n things then (n — r) things goes to original place (Here r < n) If Dn = No. of ways, if all n things goes to wrong place and Dr = No. of ways, if r things goes to wrong place If r goes to wrong places out of n, then (n — r) goes to correct places. Then Dn = nCn - r Dr 1 1 1 1 1 Where Dr r! 1 ....................... ( 1) r 1! 2! 3! 4! r! Illustration 41. A person writes letters to six friends and addresses the corresponding envelopes. in how many ways can the letters he placed in the envelopes so that (i) at least two of them are in the wrong envelopes. (ii) all the letters are in the wrong envelopes. 6 n Solution. (i ) The number of ways in which at least two of them in the wrong envelopes Cn – r Dr r 2 n n n n n Cn – 2 D2 Cn –3 D3 Cn – 4 D4 Cn –5 D5 Cn – 6 D6 Here n 6 1 1 1 1 1 1 1 1 1 6 C4 .2! 1 – 6 C3 .3! 1 – – 6 C2 .4! 1 – – 1! 2! 1! 2! 3! 1! 2! 3! 4! 1 1 1 1 1 1 1 1 1 1 1 6 C1 .5! 1 – – – 6 C0 6! 1 – – – 1! 2! 3! 4! 5! 1! 2! 3! 4! 5! 6!
  • 24. 15 40 135 264 265 719 (ii ) The number of ways is which all letters be placed in wrong envelopes 1 1 1 1 1 1 6! 1 – – – 1! 2! 3! 4! 5! 6! 1 1 1 1 720 – –120 2 6 24 720 360 –120 30 – 6 1 265 Alternative Method: (i) The number of all the possible ways of putting 6 letters into 6 envelopes is 6 !. Number of ways to place all letters correctly into corresponding envelopes = 1 and Number of ways to place one letter is the wrong envelope and other 5 letters in the write envelope = 0 (: It is not possible that only one letter goes in the wrong envelop when if 5 letters goes in the right envelope, then remaining one letter also goes in the write envelope) Hence number of ways to place at least two letters goes in the wrong envelopes =6!—0—l =6! —1 = 720 —1 =719 (ii) The number of ways I letter in 1 address envelope so that one letter is in wrong envelope=0 (Because it is not possible that only one letter goes in the wrong envelope) The number of ways to put 2 letters in 2 addressed envelopes so that all are in wrong envelopes = The number of ways without restriction — The number of ways in which all are in correct envelopes — The number of ways in which 1 letter is in the correct envelop =2!—l—0 =2—1 =1 (2){from(1)} The number of ways to put 3 letters in 3 addressed envelopes so that all are in wrong envelopes = The number of ways without restriction — The number of ways in which all are in correct envelopes — The number of ways in which 1 letter is in the correct envelope — The number of ways in which 2 letter are in correct envelope 3!—l—3C1 x l—0 (from(1),(2)} =2 (3C1 means that select one envelope to put the letter correctly) The number of ways to put 4 letters in 4 addressed envelopes so that all are in wrong envelopes The number of ways without restriction — The number of ways in which all are in correct envelopes — The number of ways in which I letter is in the correct envelope — The number of ways in which 2 letter are in correct envelopes — The number of ways in which 3 letters are in correct envelopes
  • 25. 4! –1 – 4 C1 2 – 4 C2 1 – 4 C3 0 24 –1 – 8 – 6 9 The number of ways to put 5 letters in 5 addressed envelopes so that all are in wrong envelopes = The no. of ways without restriction — The no. of ways in which all are in correct envelopes — The no. of ways in which 1 letter is in correct envelop — The no. of ways in which 2 letters are in correct envelopes — The-no, of ways in which 3 letters are in correct envelopes — The no. of ways in which 4 letters are in correct envelopes 5! –1 – 5C1 9 – 5C2 2 – 5C3 1 – 5C4 0 120 –1 – 45 – 20 –10 – 0 44 The number of ways-to put 6 letters in 6 addressed envelopes so that all are in wrong envelopes = The no. of ways without restriction — The no. of ways in which all are in correct envelopes — The no. of ways in which I letter is in the correct envelop — The no. of ways in which 2 letters are in correct envelopes — The no. of ways in which 3 letters are in correct envelopes — The no. of ways in which 4 letters are in correct envelopes — The no. of ways in which 5 letters are in correct envelopes. 6! –1 – 6 C1 44 – 6 C2 9 – 6 C3 2 – 6 C4 1 – 6 C5 0 {from (1), (2), (3), (4) & (5)} 720 –1 – 264 –135 – 40 –15 720 – 455 265. Multinomial Theorem (i) If there are 1 objects of one kind, m objects of second kind, n objects of third kind and so on; then the number of ways of choosing r objects out of these objects (i.e.,l + m + n + ...) is the coefficient of xr in the expansion of (l+x+x2+...+xl(l+x+x2+...+xm)( l+x+x2+...+xn) Further if one object of each kind is to be included, there the number of ways of choosing r objects out of these objects (i.e., 1 + m + n + ...) is the coefficient of x in the expansion of (x + x2 ± x3 + ... x’) (x + + + ... X) (x + + x3 + ... + x) (ii) If there are I objects of one kind, m object of second kind, n object of third kind and so on; then the number of possible arrangements/permutations of r objects out of these objects (i.e. 1+ m + n + ...) is the coefficient of x in the expansion of x x2 xl x x2 xm x x2 xn r! 1 ............ 1 .......... 1 ........... 1! 2! l! 1! 2! m! 1! 2! n! Illustration 42. Find the number of Combinations and Permutations of 4 letters taken from the word EXAMINATION. Solution. There are 11 letters A, A; I, I; N, N; E, X, M, T, O. 3 5 Then No. of combinations coefficient of x 4 in 1 x x 2 1 x  2 A ' s, 2 I ' s, 2 N ' s,1E ,1X ,1M ,1Tand1O 3 2 5 coefficient of x 4 in 1 x x6 3 1 x x2 3 1 x x4 1 x 8 5 7 6 coefficient of x 4 in 1 x x6 1 x 3x 2 1 x 3x 4 1 x 8 C4 0 3.7 C2 3
  • 26. and No. of Permutations 3 5 4 x x2 x coefficient of x in 4! 1 1 1! 2! 1! 3 x2 5 coefficient of x 4 in 4! 1 x 1 x 2 3 x6 3 2 3 4 5 coefficient of x 4 in 4! 1 x 1 x x2 x 1 x 1 x 8 2 4 8 x6 5 3 2 7 3 4 6 coefficient of x 4 in 4! 1 x 1 x x 1 x x 1 x 8 2 4 8 37 3 4! C4 0 . C2 2 4 8.7.6.5 3 7.6 3 24 . 1.2.3.4 2 1.2 4 8. 7. 6. 5 6 3. 7. 6 6. 3 1680 756 18 Alternative Method : There are 11 letters: A, A;I, I;N, N;E, X, M, T, O. The following cases arise: Case I : All letters different : The required number of choosing 4 different letters from 8 different (A, I, N, E, X, M, T, 0) types of the letters = 2454 8 8.7.5.6 C4 70 1.2.3.4 8 and No. of Permutations = 8P4 = 8.7.6.5 = 1680 Case II : Two alike of one type & Two alike of another type 2A’s, 2I’s, or 2I’s, 2N’s, or 2N’s, 2A’s.: This must be = 18 Case III: Two alike and Two different: This must be 2A’s or 21’s or 2N’s and for each case 7 different letters.
  • 27. = 756 From Case I,II and III, The required No. of Combinations =70+3+63 and No. of Permutations = 1680 + 18 + 756 =2454. How to find number of Solutions of the Equation If the equation 2 3 ... q n .....(1) (i ) If zero included then no. of solutions of (1) coefficient of x n in 1 x x 2 ... 1 x 2 x 4 ... 1 x3 x 6 ... ... 1 x q x 2 q ... –1 –1 –1 –1 coefficient of x n in 1 – x 1 – x2 1 – x3 ... 1 – x q (ii ) If zero exluded then the no. of solutions of (1) coefficient of x n in x x 2 x 3 .... x 2 x4 x 6 ... x 3 x6 x 9 ... .... x q x2q ... –1 2 –1 3 –1 q –1 coefficient of x n in x1 2 3 ... q 1– x 1– x 1– x ... 1 – x q q 1 n– –1 –1 –1 –1 coefficient of x 2 in 1 – x 1 – x2 1 – x3 ... 1 – x q Illustration 43. Find the number of non negative integral solutions of x1 x2 x3 4 x4 20 . Solution. Number of non negative integral solutions of the given equation –1 –1 –1 –1 coefficient of x 20 in 1 – x 1– x 1– x 1 – x4 –3 –1 coefficient of x 20 in 1 – x 1 – x4 coefficient of x in (1 C1 x 4 C2 x 2 20 3 5 C3 x 3 6 C4 x 4 ... 10 C8 x8 ... 14 C12 x12 .. 18 .... C16 x16 ... 22 C20 x 20 ...) 1 x4 x8 x12 x16 x 20 ... 6 10 14 18 22 1 C4 C8 C12 C16 C20 6 10 14 18 22 1 C2 C2 C2 C2 C2 6. 5 10. 9 14. 13 18. 17 22. 21 1 1. 2 1. 2 1. 2 1. 2 1. 2 1 15 45 91 153 231 536 Illustration 44. Find the number of positive unequal integral solution of the equation x + y + z + w = 20; Solution. We have x y z w 20 assume x y z w, Here x, y, z , w 1 now let x x1 , y – x x2 , z – y x3 and w – z x4 x x1 , y x1 x2 , z x1 x2 x3 , w x1 x2 x3 x4 from (1), 4 x1 3 x2 2 x3 x4 20, then x1 , x2 , x3 , x4 1  4 x1 3 x2 2 x3 x4 20 ......(2) Number of positive integral solutions of (2) –1 –1 –1 –1 coefficient of x 20–10 in 1 – x 4 1 – x3 1 – x2 1– x
  • 28. x6 x8 x10 x9 x4 x6 x8 x10 x7 x9 x10 x8 x10 )(1 x x 2 x3 x4 x5 x6 x7 x8 x9 x10 ) 1 1 1 1 1 1 1 1 1 1 1 1 1 1 1 1 1 1 1 1 1 1 1 23 Number of Rectangles and Squares n n (i) Number of rectangles of any size in a square of n x n is r 3 and number of squares of any size is r2 r 1 r 1 np (ii) In a rectangle of n x p (n <p) number of rectangles of any size is (n 1)( p 1) and number of squares of any 4 n size is (n 1 r )( p 1 r ) r 1 Illustration 45. Find the number of rectangles excluding squares from a rectangle of size 9 × 6. Solution. Here n =6 and p =9. No. of rectangles excluding square 6 6.9 (6 1)(9 1) – (7 – r )(10 – r ) 4 r 1 6 945 – (70 –17r r 2 ) r 1 945 –154 791. Exponent of Prime p inn! Since exponent of prime p in n is denoted by Ep (n !). where p is a prime number and n is a natural number. Then the n n last integer amongst 1, 2, 3, ...,(n — 1), n which is divisible by p is p, where denotes the greatest integer. p p
  • 29.  [ x] x E p (n !) E p (1.2.3.....(n –1).n) n E p p.2 p.3 p.....(n –1) p. p p because the remaining natural numbers from 1 to n are not divisible by p. n n E p 1.2.3.... p p n Now the last integer amongs 1, 2,3...., which is divisible by p is p n. p n n n E p p.2 p.3 p.... p p p2 p p2 n because the remaining natural numbers from 1 to are not divisible by p. p n n n E p 1.2.3.... p p2 p2 similarly, we get n n n n E p (n !) .... p p2 p3 ps Where s is the largest natural number such that p s n ps 1 Illustration 46. Find the exponent of 3 in 100!. Solution: In terms of prime factors 100 ! can be written as 2a. 3b 5c. 7d……………………. 100 100 100 100 Now, E3 (100!) 3 32 33 34 =33 + 11+3+1=48 Hence, the exponent of 3 in 100 ! is 48 ( 100 ! = 2a. 348 .5c .7d ) 33 33 33 33 33 E2 (33!) 2 22 23 24 25 = 16+8+4+2+1 = 31 Hence, the exponent of 2 in 33 ! in 31. Now 33 ! is divisible by 231 which is also divisible by 219 largest value of n is 31. Illustration 47. Find the number of zeros at the end of 100!? Solution: In terms of prime factors 100 ! can be written as 2a. 3b .5c. 7d 100 100 100 100 100 100 Now, E2 (100!) 2 22 23 24 25 26
  • 30. 50 25 12 6 3 1 97 100 100 and E5 (100!) 5 52 20 4 24 100! 297.3b.524.7 d ... 273.3b.(2 5) 24 .7 d ... 273.3b.(10) 24 .7 d ... Hence number of zeros at the end of 100! is 24. OR Exponent of 10 in 100! min (97, 24) 24. Important Results to Remember (i) n straight lines are drawn in the plane such that no two lines are parallel and no three lines are concurrent. Then the number of parts into which these lines divide the plane is equal to 1+ n Illustration 48. Six straight lines are drawn in the plane such that no two lines are parallel and no three lines are concurrent. Then find the number of parts into which these lines divide the plane. Solution. No. of parts of the plane = 1 + 6 6.7 =1+ = 22 1.2 (ii) The sum of the digits in the unit place of all numbers formed with the help of a1 ,a2………… an taken all at a time is = (n — 1)! (a1 ,a2………… an) (repetition of digits not allowed) Illustration 49. Find the sum of the digits in the unit place of all numbers formed with the help of 3, 4, 5, 6 taken all at a time. Solution. Sum of the digits in the unit place = (4—1)!(3+4+5+6) = 6.18 = 108. (iii) The sum of all digit numbers that can be formed using the digits a1 ,a2………… an (repetition of digits not allowed) is (10n 1) (n 1)!(a1 a2 ............... an ) 9 Illustration 50: Find the sum of all five digit numbers that can he formed using the digits 1, 2,3, 4 and 5 (repetition of digits not allowed). (105 1) 9
  • 31. Solution. Required sum= (5—1)! (1 +2+3+4+5) = 24.15. 11111 = 3999960. Alternative Method: Since one of the numbers formed with the 5 digits a , b , c , d, e is 104a+ 103b + 102c+ l0d+e; Hence 104a will occur altogether in 4 ! Ways similarly each of 104b, 104c, 104d, 104e will occur in 4 ! Ways. Hence, if all the numbers formed with the digits be written one below the other, thus 104.a 103.b 102.c 10.d e 104.b 103.c 102.d 10.e a 104.c 103.d 102.e 10.a b 104.d 103.e 102.a 10.b c 104.e 103.a 102.b 10.c d Hence the required sum 4! (a b c d e) (104 103 102 10 1) 4! (1 2 3 4 5) (11111) 3999960. (iv) If there are n rows, I row has 1 squares. II row has 2 . squares, III row has 3 squares, and so on. If we placed Xs in the squares such that each row contain at least one X. The number of ways = coefficient of x in 1 C1 x 1 C2 x 2 ............ 1 C 1x 1 2 C1 x 2 C2 x 2 ............ 2 C 2x 1 ( 3 C1 x 3 C2 x 2 ..... ....... 3 C 3x 3) Illustration 51. Six X’s have to be placed in the squares of the figure below, such that each row contains at least one X. In how many different ways can this be done? Solution: The required no. of ways = coefficient of x6 in ( 2 C1 x 2 C2 x 2 )( 4 C1 x 4 C2 x 2 4 C3 x 3 4 C4 x 4 )( 2 C1 x 2 C2 x 2 ) = coefficient of x3 in (2 + x)2 (4 + 6x + 4x2 + x3)
  • 32. = coefficient of x3 in (4 + 4x + x2) (4 + 6x + 4x2 + x3) = 4+16+6 = 26 Alternative Method In the given figure there are 8 squares and we hay to place 6X ‘s this can be done in X X X X X X 8 8 8.7 i.e. C6 C2 28 ways 1.2 But these include the possibility that either headed row or lowest row may not have any X. These two possibilities are to be excluded. Required number of ways = 28 — 2 = 26 Solved Examples Example 1: A letter lock consists of three rings each marked with 10 different letters. In how many ways is it possible to make an unsuccessful attempt to open the block? Solution: Two rings may have same letter at a time but same ring cannot have two letters at time, therefore, we must proceed ring wise. Each of the three rings can have any one of the 10 different letters in 10 ways. Therefore Total number of attempts = 10 × 10 × 10 = 1000. But out of these 1000 attempts only one attempt is successful. Therefore Required number of unsuccessful attempts = 1000 - 1 = 999.
  • 33. Example 2: Find the total number of signals that can be made by five flags of different colour when any number of them may be used in any signal. Solution: Case I : When only one flag is used. No. of signals made = 5P1 = 5. Case II : When only two flag is used. Number of signals made = 5P2 = 5.4 = 20. Case III : When only three flags are used. Number of signals is made = 5P3 = 5.4.3 = 60. Case IV : When only four flags are used. Number of signals made = 5P4 = 5.4.3.2 = 120. Case V : When five flags are used. Number of signals made = 5P5 = 5! = 120. Hence, required number = 5 + 20 + 60 + 120 + 120 = 325. Example 3: Prove that if each of the m points in one straight line be joined to each of the n points on the other straight line, the excluding the points on the given two lines. Number of points of intersection of these lines is 1/4 mn (m-1(n-1). Solution: To get one point of intersection we need two points on the first line and two points on the second line. These can be selected out of n-points in nC2 ways and for m points in mC2 ways. 2n 1 2n 1 2n 1 2n 1 Therefore Required number = Now C0 C2 n 1 1 C1 C2 n etc......... m m 1 n n 1 = . 2! 2! = 1/4 m n (m - 1)(n - 1) Example 4: There are ten points in a plane. Of these ten points four points are in a straight line and with the exception of these four points, no other three points are in the same straight line. Find (i) the number of straight lines formed. (ii) the number of triangles formed. (iii) the number of quadrilaterals formed by joining these ten points. Solution: (i) For straight line, we need 2 points No. of point selected out No. of points selected out No. of straight line of 4 collinear points of remaining 6 points formed 4 0 2 C0 × 6C2 = 15 4 1 1 C1 × 6C1 = 24